Download 6A - UAB School of Optometry

Survey
yes no Was this document useful for you?
   Thank you for your participation!

* Your assessment is very important for improving the workof artificial intelligence, which forms the content of this project

Document related concepts

Biofilm wikipedia , lookup

Horizontal gene transfer wikipedia , lookup

Infection control wikipedia , lookup

Skin flora wikipedia , lookup

Trimeric autotransporter adhesin wikipedia , lookup

Gastroenteritis wikipedia , lookup

Molecular mimicry wikipedia , lookup

Bacteria wikipedia , lookup

Infection wikipedia , lookup

Marine microorganism wikipedia , lookup

Disinfectant wikipedia , lookup

Magnetotactic bacteria wikipedia , lookup

Hospital-acquired infection wikipedia , lookup

Traveler's diarrhea wikipedia , lookup

Triclocarban wikipedia , lookup

Bacterial taxonomy wikipedia , lookup

Human microbiota wikipedia , lookup

Bacterial cell structure wikipedia , lookup

Neisseria meningitidis wikipedia , lookup

Bacterial morphological plasticity wikipedia , lookup

Transcript
Chapter 1: Bacterial Structure, Physiology, and Classification
A.) Compare and contrast the properties of eukaryotes vs. prokaryotes. (p.11, Fig. 3.1)
Prokaryotes
No nuclear membrane – single circular
chromosome
Extrachromosomal DNA present – plasmid
Few organelles
Cytoplasmic membrane – respiration,
secretion and macromolecular synthesis
Cell wall has peptidoglycan – except in
mycoplasma
Sterols absent
70S ribosomes
Eukaryotes
Membrane bound nucleus – many
chromosomes
Extrachromosomal DNA in organelles
Several organelles
Lacks these functions
No peptidoglycan
Sterols usually present
80S ribosomes
B.) Compare and contrast cell wall components in gram-positive and gram-negative
bacteria. (p. 12-24)
GRAM (+):
 Thick peptidoglycan – 10 to 100 nm
 Wall teichoic acids (WTA) – covalently linked to PG
 Lipoteichoic acids (LTA) membrane anchored, not covalently linked
 WTA and LTA – ion binding, charge maintenance, membrane integrity, adherence, anchor proteins
GRAM (-):
 Single PG layer – but NOT continuous
 No WTA or LTA
C.) Understand structure and function of each bacterial major ultrastructural component:
chromosome, plasmid, ribosome, inner (cytoplasmic) membrane, outer membrane, mesosome, teichoic acid,
peptidoglycan, lipopolysaccharide, capsule, pili, flagella, endospores. (p.12-22).
Bacterial Chromosomes
• Single, circular, double-stranded DNA
(exception - borrelia = linear)
• Haploid (1 to 4 copies depending on growth rate) (compare this to eukaryotes that are diploid)
• 600 to 4500 kb* in size. Smaller = more
dependent on host/environment
• Up to 1 mm in length; supercoiled
• Contained in nucleoid
* ~1 kb/gene
Plasmids
 Replicate in cytoplasm,
 independent of chromosome.
 Usually circular (borrelia = linear)
 Few to several hundred kb.
 Conjugative (F, R), antibiotic resistance,
metabolic, virulence
Cytoplasmic Membrane









Lipid bilayer
Permeability barrier
Active transport
Electron transport
Oxidative phosphorylation
Photosynthesis
Affected by antibacterials
Detergents
Polymyxins (damage PE containing membranes)
 Ionophores (disrupt membrane potential)
Outer membrane (OM) (seen in gram negative)
 blocks entry of large molecules (>800 Da). Not typical lipid bilayer.
 Attached to PG by lipoprotein
 Lipopolysaccharide (LPS) - forms outer leaflet of OM
 OM proteins - transport, porins
Mesosome: no answer was given
Teichoic acid (both are gram-positives)
 Wall teichoic acids (WTA) - repeating units of phosphodiester linked glycerol or ribitol backbone + side
chains (D-ala, glucose).
 Covalently linked to PG.
 Lipoteichoic acids (LTA) - membrane-anchored, structure may differ from WTA
 LTA and WTA - ion binding, charge maintenance, membrane integrity, adherence, anchor proteins
Peptidoglycan
 Backbone of N-acetyl glucosamine and Nacetyl muramic acid (transglycosylation)
 Cross-linked by peptide bridges at MurNAc (transpeptidation)
 β-lactam antibiotics prevent crosslinking
 Lysozyme (hydrolase) - cleaves backbone (M-G)
Lipopolysaccharide (LPS)
 Endotoxin - toxic shock (OM)-Lipid A --- core polysaccharide --- O Ag
 toxic properties
 Toxic shock, fever, leukopenia, hypotension, acidosis, DIC, death
 varies with species
 polysaccharide varies with strain 3 - 4 sugars/repeat Up to 25 repeats serotyping
Capsules - polysaccharide or protein
 Antiphagocytic (block C3b deposition or recognition, attachment
Pili - protein. Shorter, narrower than flagella.
 Common - peritrichous; attachment
 F (sex) - single; gene transfer (conjugation; gram -)
Flagella - protein. Rotates to propel cell.
 Motility, chemotaxis, virulence;
 Peritrichous - all around
 Polar - one end
 Bipolar - both ends
Endospores - dehydrated cells; Clostridium,
 Bacillus species (gram +)
D.) Describe the procedure for Gram-stain and explain the purpose of each reagent. (p.12)
After preparation of Smear,
1. Flood slide with crystal violet solution (adheres to the peptidoglycan of the cell wall) and allow it to remain on the
slide for 60 seconds. Rinse slide with water.
2. Flood slide with iodine solution (serves as a mordant by increasing the affinity of the dye for the bacterial cell) and
allow to remain for one minute. Rinse slide with water.
3. Rinse slide with 95% alcohol (washes away the non-adherent to cell wall CV solution) until the solvent flows
colorlessly from the slide (about 2 seconds).
4. Counterstain the bacterial smear on the slide with safranin (stains the gram negative bacteria which have thin
peptidoglycan walls that have been rinsed clean of the crystal violet) for 30 seconds, then wash slide with water.
E.) Explain the structural differences of the mycobacterial cell wall from other bacteria that cause it to be
“acidfast.” (p. 18)
Mycobacteria have a peptidoglycan layer (slightly different structure), which is intertwined with and covalently
attached to an arabinogalactan polymer and surrounded by a waxlike lipid coat of mycolic acid (large alpha-branced
beta-hydroxy acids), cord factor, waxD and sulfolipids. These bacteria are described as acid-fast staining. This coat
is responsible for virulence and is antiphagocytic. Corynebacterium and Nocardia organisms also produce mycolic
acid lipids. The mycoplasmas are also exception in that they have no peptidoglycan cell wall and they incorporate
steroids from the host into their membranes.
F.) Explain the process by which peptidoglycan is synthesized. (p. 18-19)
Peptidoglycan synthesis occurs in four steps (see Figure 3-8). First, inside the cell, glucosamine is enzymatically
converted into MurNAc and then energetically activated by a reaction with uridine triphosphate (UTP) to produce
uridine diphosphate-N-acetylmuramic acid (UDP-MurNAc). Next, the UDP-MurNAc-pentapeptide precursor is
assembled in a series of enzymatic steps. Second, the UDP-MurNAc pentapeptide is attached to the bactoprenol
"conveyor belt" in the cytoplasmic membrane through a pyrophosphate link, with the release of uridine
monophosphate (UMP). GlcNAc is added to make the disaccharide building block of the peptidoglycan. Some
bacteria (e.g., S. aureus) add a pentaglycine or another chain to the diamino amino acid at the third position of the
peptide chain to lengthen the cross-link. Third, the bactoprenol molecule translocates the disaccharide:peptide
precursor to the outside of the cell. The GlcNAc-MurNAc disaccharide is then attached to a peptidoglycan chain,
using the pyrophosphate link between itself and the bactoprenol as energy to drive the reaction by enzymes called
transglycosylases. The pyrophosphobactoprenol is converted back to a phosphobactoprenol and recycled. Fourth,
outside the cell but near the membrane surface, peptide chains from adjacent glycan chains are cross-linked to each
other by a peptide bond exchange (transpeptidation) between the free amine of the amino acid in the third position of
the pentapeptide (e.g., lysine), or the N-terminus of the attached pentaglycine chain, and the d-alanine at the fourth
position of the other peptide chain, releasing the terminal d-alanine of the precursor. This step requires no additional
energy because peptide bonds are "traded."
The cross-linking reaction is catalyzed by membrane-bound transpeptidases. Related enzymes, ddcarboxypeptidases, remove unreacted terminal d-alanines to limit the extent of cross-linking. The transpeptidases and
carboxypeptidases are called penicillin-binding proteins (PBPs) because they are targets for penicillin and other βlactam antibiotics. Penicillin and related β-lactam antibiotics resemble the "transition state" conformation of the dALA-d-ALA substrate when bound to these enzymes. Different PBPs are used for extending the peptidoglycan,
creating a septum for cell division, and curving the peptidoglycan mesh (cell shape).
The peptidoglycan is constantly being synthesized and degraded. Autolysins such as lysozyme are important for
determining bacterial shape. Inhibition of synthesis or the cross-linking of the peptidoglycan does not stop the
autolysins, and their action weakens the mesh and the bacterial structure and leads to lysis and cell death. New
peptidoglycan synthesis does not occur during starvation, which leads to a weakening of the peptidoglycan and a loss
in the dependability of the Gram stain.
G.) Explain the process and purpose of spore formation and name the two main genera of bacterial pathogens
that produce spores. (p. 22-24)
Bacteria form spores as a form of survival. It allows them to dehydrate and go into a dormant stage until a host
comes along for it to enter. The two main bacterial pathogens that form spores are Clostridium and Bacillus.
H.) Describe how mycoplasmas are unique from other bacteria and how these differences are responsible for
their morphology and life cycle. (p.18)
Mycoplasmas are bacterial exceptions because they have no peptidoglycan cell wall and they incorporate steroids
from the host into their membranes.
I.) Explain how the presence of a capsule is important as a bacterial virulence factor and provide examples of
clinically important bacteria that are encapsulated. (p. 17)
The capsule provides protection to the bacteria and is therefore an important bacterial virulence factor. It makes up
the outer-most layer and consists of either polysaccharides of proteins. Its main function is to be antiphagocytic and
block C3b deposition or recognition and attachment. This means that the capsule either prevent s the complement
from binding to the cell wall, or if the complement does manage to attach, it will prevent the phagocyte receptors
from binding to the complement, therefore preventing phagocytosis. Some examples of bacteria with capsules are K.
pheumoniae, B. pertussis, N. meningitides.
J.) Understand the processes of DNA replication, mRNA transcription and translation and the steps involved
in each. (p. 30-31)
The synthesis of the purine nucleotides (e.g., adenosine monophosphate and guanosine monophosphate) begins with
ribose-5-phosphate formed as a product of the pentose phosphate pathway. The bicyclic purine ring system is
constructed in a stepwise fashion on the sugar phosphate moiety. The product of this series of reactions is the purine
nucleotide, inosine monophosphate, which may then be converted to guanosine or adenosine monophosphate. In
contrast, the pyrimidine nucleotides are produced by the synthesis of the pyrimidine, orotate, which is then attached
to the ribose phosphate, forming orotidine monophosphate. This nucleotide may then be converted to cytidine
monophosphate or uridine monophosphate. The corresponding deoxyribonucleotides for use in DNA are obtained by
removal of the hydroxyl at the 2' carbon atom of the sugar portion of the ribonucleotide. Production of thymine, a
unique and required nucleotide for DNA, requires the tetrahydrofolate pathway, which makes this pathway a target
for antibiotic action.
TRANSCRIPTION
The information carried in the genetic memory of the DNA is transcribed into a useful messenger RNA (mRNA) for
subsequent translation into protein. RNA synthesis occurs in a manner similar to that of DNA replication, using a
specific DNA-dependent RNA polymerase.
The process begins when sigma factor recognizes a particular sequence of nucleotides in the DNA (the promoter; see
Chapter 5) and binds tightly to this site. Promoter sequences occur just before the start of the DNA that actually
encodes a protein. Sigma factors bind to these promoters to provide a docking site for the RNA polymerase. Some
bacteria encode several sigma factors to allow transcription of a group of genes under special conditions, such as heat
shock, starvation, special nitrogen metabolism, or sporulation. Once the polymerase has bound to the appropriate site
on the DNA, RNA synthesis proceeds with the sequential addition of ribonucleotides complementary to the sequence
in the DNA. Once an entire gene or group of genes (operon; see Chapter 5) has been transcribed, the RNA
polymerase dissociates from the DNA, a process mediated by signals within the DNA. The bacterial, DNA-
dependent RNA polymerase is inhibited by rifampin, an antibiotic often used in the treatment of tuberculosis. The
transfer RNA (tRNA), which is used in protein synthesis, and ribosomal RNA (rRNA), a component of the
ribosomes, are also transcribed from the DNA.
TRANSLATION
Translation is the process by which the language of the genetic code, in the form of mRNA, is converted (translated)
into a sequence of amino acids, the protein product. For the purposes of translation, the nucleotide sequence of the
mRNA is divided into groups of three consecutive nucleotides. Each set of three nucleotides is known as a codon and
encodes a particular amino acid. Because there are four different nucleotides, 43, 64 combinations of 3 are possible,
each codon coding for only a single amino acid (or termination signal). However, because there are only 20 amino
acids, each may be encoded by more than one triplet codon. This feature is known as the degeneracy of the genetic
code and may function in protecting the cell from the effects of minor mutations in the DNA or mRNA. Each tRNA
molecule contains a three-nucleotide sequence complementary to one of the codon sequences. This tRNA sequence is
known as the anticodon; it allows base pairing and binds to the codon sequence on the mRNA. Attached to the
opposite end of the tRNA is the amino acid that corresponds to the particular codon-anticodon pair.
The process of protein synthesis (Figure 4-9) begins with the binding of the 30S ribosomal subunit and a special
initiator tRNA for formyl methionine (fmet) at the methionine codon (AUG) start codon to form the so-called
initiation complex. The 50S ribosomal subunit binds to the complex to initiate mRNA synthesis. The ribosome
contains two tRNA binding sites, the A (aminoacyl) site and the P (peptidyl) site, each of which allows base pairing
between the bound tRNA and the codon sequence in the mRNA. The tRNA corresponding to the second codon
occupies the A site. The amino group of the amino acid attached to the A site forms a peptide bond with the carboxyl
group of the amino acid in the P site in a reaction known as transpeptidation. This process leaves the tRNA in the P
site uncharged (i.e., without an attached amino acid), allowing it to be released from the ribosome. The ribosome
then moves down the mRNA exactly three nucleotides, thereby transferring the tRNA with attached nascent peptide
to the P site and bringing the next codon into the A site. The appropriate charged tRNA is brought into the A site, and
the process is then repeated. Translation continues until the new codon in the A site is one of the three termination
codons, for which there is no corresponding tRNA. At that point the new protein is released to the cytoplasm and the
translation complex may be disassembled, or the ribosome shuffles to the next start codon and initiates a new protein.
The ability to shuffle along the mRNA to start a new protein is a characteristic of the 70S bacterial but not of the 80S
eukaryotic ribosome. This has implications for the synthesis of proteins for some viruses.
The process of protein synthesis by the 70S ribosome represents an important target of antimicrobial action. The
aminoglycosides (e.g., streptomycin and gentamicin) and the tetracyclines act by binding to the small ribosomal
subunit and inhibiting normal ribosomal function. Similarly, the macrolide (e.g., erythromycin) and lincosamide
(e.g., clindamycin) groups of antibiotics act by binding to the large ribosomal subunit.
K.) Describe the events that occur in each phase of bacterial growth. (p.33, Fig. 4-11)
1. Lag phase – bacteria are actively metabolizing and preparing for active growth
2. Log phase – exponential growth of bacteria – growth rate depends on type of bacteria and the growing
conditions
3. Stationary phase – slowed metabolic activity and slow growth due to a decrease in nutrients or toxic products.
4. Death phase – exponential decline and loss of viability – either natural or induced by detergent, antibiotics,
heat, radiation or chemicals
L.) Explain the difference between oxidation and fermentation and give examples of bacteria which are
fermenters, oxidizers or asaccharolytics. (p. 28-29; lab syllabus)
Oxidation (non-fermenters)
 Carried out by obligate aerobes and facultative anaerobes
 Efficient energy-generating process in which molecular oxygen is the final electron receptor
 require catalase and superoxide dismutase to degrade toxic O2
 ex. Bacillus, P. aeruginosa
Fermentation
 an anaerobic process carried out by both obligate and facultative anaerobes
 electron acceptor is an organic compound
 less sufficient energy yielding process than oxidation because the substrate is not completely reduces and
therefore all of the energy is not released
 ex. Clostridium perfringens, Enterobacteriaceae
Asaccharolytics
 use substrates other than carbohydrates for metabolic energy
M.) Explain the importance of determining genetic relatedness of bacteria in epidemiology and infection
control. (lecture outline)
Determining the genetic relatedness of different types (or strains of the same type) of bacteria is important in
epidemiology and infection control. Many different bacteria contain similar surface antigens. For example, knowing
which bacteria express a given surface antigen allows grouping of organisms for potential antibiotic production. On
the other hand, a single species may have many potential polysaccharide capsules that it can produce. Being able to
determine which capsule is present on that species during an outbreak of infection is important for stopping the
outbreak before it becomes an epidemic.
(Note: I got this next part off the internet—not sure if it will help.)
Particularly for epidemiological purposes, clinical microbiologists must distinguish strains with particular traits from
other strains in the same species. For example, serotype O157:H7 E coli are identified in stool specimens because of
their association with bloody diarrhea and subsequent hemolytic uremic syndrome. Below the species level, strains
are designated as groups or types on the basis of common serologic or biochemical reactions, phage or bacteriocin
sensitivity, pathogenicity, or other characteristics. Many of these characteristics are already used and accepted:
serotype, phage type, colicin type, biotype, bioserotype (a group of strains from the same species with common
biochemical and serologic characteristics that set them apart from other members of the species), and pathotype (e.g.,
toxigenic Clostridium difficile, invasive E coli, and toxigenic Corynebacterium diphtheriae).
N.) Explain the differences, advantages, and disadvantages among phenotypic, analytic, and genotypic
classification of bacteria and provide examples of each approach. (p.8; lecture outline)
Phenotypic Classification—This type of classification is based on morphological characteristics, growth
characteristics, antigen susceptibility, and biochemical characteristics (basically everything we do in lab). Its
advantages are ease of testing and relatively low cost making them highly useable for lesser equipped labs. The
disadvantages are that phenotypic properties of bacteria are often highly variable and the tests require a lot of labor.
Examples include testing for growth in the presence/absence of oxygen, ability to ferment/oxidize glucose, etc.
Analytic Classification—In analytical taxonomy (also called numerical taxonomy) many biochemical,
morphological, and cultural characteristics, as well as susceptibilities to antibiotics and inorganic compounds, are
used to determine the degree of similarity between organisms. In some cases, certain characteristics may be
weighted more heavily. The major advantage to this type of classification is that it accounts for many different
characteristics when comparing organisms. One disadvantage to this classification is that when this approach is the
only basis for defining a species, it is difficult to know how many and which tests should be chosen; whether and
how the tests should be weighted; and what level of similarity should be chosen to reflect relatedness at the genus
and species levels. Other types of errors may occur when species are classified solely on the basis of phenotype.
Basically the example of this type is to run the data on a computer model and come up with a coefficient which can
be graphed and plotted to compare relatedness.
Genotypic Classification—This type of classification is based on the DNA or RNA sequences of specific bacteria.
This is the gold standard of classification in that bacteria are classified by their entire genomes, not just those genes
which they actively express. The advantages of genotypic classification are high discriminatory power and the
ability to apply the same tests to many different types of bacteria. These tests, however, are much more expensive
than the others and are highly time and labor consuming.
O.) Explain the specific advantages and reasons that characterization of rRNA is a useful means for
determining genetic relatedness of bacteria. (lecture outline)
The best way to classify bacteria is by the rRNA sequence. The rRNA is critical for protein synthesis and binds to
the initiation site on the mRNA in translation. The DNA that encodes rRNA is highly conserved among bacteria of
common ancestry and is therefore the basis of the phylogenetic trees.
Chapter 2: Bacterial Pathogenesis
A.) Explain the differences between microbial colonization and infection and give examples of each process.
(p. 83)
It is important to understand the distinction between colonization and disease (Box 9-1). (Note: many people use the
term infection inappropriately as a synonym for both terms.) Organisms that colonize humans (whether for a short
period such as hours or days [transient] or permanently) do not interfere with normal body functions. In contrast,
disease occurs when the interaction between microbe and human leads to a pathologic process characterized by
damage to the human host. This process can result from microbial factors (e.g., damage to organs caused by the
proliferation of the microbe or the production of toxins or cytotoxic enzymes) or the host's immune response to the
organism (e.g., the pathology of several acute respiratory syndrome [SARS] coronavirus infections is primarily
caused by the patient's immune response to the virus).
B.) Understand the differences between strict pathogens and opportunistic pathogens; be able to give specific
examples of each and describe host conditions that are favorable for opportunistic infection. (p. 83)
Opportunistic pathogens- bacteria that are commonly found in the environment and cause disease only in an
individual whose defenses are somewhat debilitated. Ex Candida and Pseudomonas
Strict Pathogens- Pathogens that are so virulent, healthy people get sick! Strept, Flu, Etc.
C.) Describe which anatomic locations in the human body contain normal flora versus those locations which
are normally sterile and the major types of bacteria that comprise the normal flora in each of these sites. (p.
84-86)
Mouth, Oropharynx and Nasopharynx-Anaerobes- peptostreptococcus, Veillonella, Actinomyces, and
Fusobacterium. Aerobic- Strep. Haemophilius and Neisseria spp.
Ear- Coagulase-Negative Staphylococcus
Eye- Coagulase negative Staph. As well as most ocganisms found in the nasopharnyx
Lower Respiritory tract- Sterile
GI- no answer was given.
Esophagus: Oropharyngeal bacteria and yeast, as well as the bacteria that colonize the stomach, can be isolated from
the esophagus; however, most organisms are believed to be transient colonizers that do not establish permanent
residence
Stomach: Small numbers of Acid tollerent bacteria such as Lactobacillus and Strept. Spp.
Small Intestine: the small intestine is colonized with many different bacteria, fungi, and parasites. Most of these
organisms are anaerobes, such as Peptostreptococcus, Porphyromonas, and Prevotella.
Large Intestine: More bacteria are in the large intestine than anywhere else in the human body. The most common
bacteria include Bifidobacterium, Eubacterium, Bacteroides, Enterococcus, and the Enterobacteriaceae family. E.
coli is present in virtually all humans from birth until death.
Anterior Urethra- The commensal population of the urethra consists of a variety of organisms, with lactobacilli,
streptococci, and coagulase-negative staphylococci the most numerous.
Vagina: The microbial population of the vagina is more diverse and is dramatically influenced by hormonal factors.
Newborn girls are colonized with lactobacilli at birth, and these bacteria predominate for approximately 6 weeks.
After that time, the levels of maternal estrogen have declined, and the vaginal flora changes to include staphylococci,
streptococci, and Enterobacteriaceae. When estrogen production is initiated at puberty, the microbial flora again
changes. Lactobacilli reemerge as the predominant organisms, and many other organisms are also isolated, including
staphylococci (S. aureus less commonly than the coagulase-negative species), streptococci (including group B
Streptococcus), Enterococcus, Gardnerella, Mycoplasma, Ureaplasma, Enterobacteriaceae, and a variety of anaerobic
bacteria.
Cervix: the cervix is not normally colonized with bacteria
Skin: Although many organisms come into contact with the skin surface, this relatively hostile environment does not
support the survival of most organisms (Box 9-5). Gram-positive bacteria (e.g., coagulase-negative Staphylococcus
and, less commonly, S. aureus, corynebacteria, and propionibacteria) are the most common organisms found on the
skin surface. Clostridium perfringens is isolated on the skin of approximately 20% of healthy individuals, and the
fungi Candida and Malassezia are also found on skin surfaces, particularly in moist sites. Streptococci can colonize
the skin transiently, but the volatile fatty acids produced by the anaerobe propionibacteria are toxic for these
organisms. Gram-negative rods do not permanently colonize the skin surface (with the exception of Acinetobacter
and a few other less common genera) because the skin is too dry.
D.) Describe the beneficial roles of normal flora in the host-microorganism ecological relationship (pg 83-87)
Normal flora is beneficial in that some produce some vitamins naturally, such as Vitamin K, which aids in blood
clotting. Normal flora also occupies niches so that pathogens can not invade in those areas.
Here is some info from the internet:
1. The normal flora synthesize and excrete vitamins in excess of their own needs, which can be absorbed
as nutrients by the host. For example, enteric bacteria secrete Vitamin K and Vitamin B12, and lactic acid
bacteria produce certain B-vitamins.
2. The normal flora prevent colonization by pathogens by competing for attachment sites or for essential
nutrients. This is thought to be their most important beneficial effect, which has been demonstrated in the
oral cavity, the intestine, the skin, and the vaginal epithelium.
3. The normal flora may antagonize other bacteria through the production of substances which inhibit or
kill nonindigenous species. The intestinal bacteria produce a variety of substances ranging from relatively
nonspecific fatty acids and peroxides to highly specific bacteriocins, which inhibit or kill other bacteria. 4.
The normal flora stimulate the development of certain tissues, i.e., the caecum and certain lymphatic
tissues (Peyer's patches) in the GI tract. The caecum of germ-free animals is enlarged, thin-walled, and fluidfilled, compared to that organ in conventional animals.
5. The normal flora stimulate the production of cross-reactive antibodies. Since the normal flora behave
as antigens in an animal, they induce an immunological response, in particular, an antibody-mediated immune
(AMI) response. Low levels of antibodies produced against components of the normal flora are known to
cross react with certain related pathogens, and thereby prevent infection or invasion. Antibodies produced
against antigenic components of the normal flora are sometimes referred to as "natural" antibodies.
E.) Explain how prolonged hospitalization or antibiotic therapy can affect the composition of normal flora.
The normal gastrointestinal flora, which includes E coli and, frequently, other coliforms and Proteus species in small
numbers, is important in preventing disease through bacterial competition. Prolonged antibiotic therapy compromises
this defense mechanism by reducing susceptible components of the normal flora, permitting nosocomial coliform
strains or other bacteria to colonize or overgrow. Prolonged hospitalization can affect the normal flora too, in that
patients have a low immune response to begin with and are very susceptible to what may be minor pathogens. This
can be made worse if health care workers do not wash their hands after each patient.
F.) Describe the clinical manifestations of endotoxin shock and mechanisms responsible for these
manifestations. (p. 197; Fig. 19-2)
Endotoxin-virulence factor shared among all aerobic and some anaerobic gram - bacteria. This toxicity resides in the
lipid A component of LPS, which is released upon cell death and lysis. Many of the systemic manifestations of gram
- infections are initiated by endotoxin. Endotoxin-Mediated Toxicity: Fever, Leukopenia followed by leukocytosis,
activation of complement, thrombocytopenia, DIC, decreased peripheral circulation and perfusion to major organs,
shock, death
G.) Describe the similarities and differences between exotoxins and endotoxins, including structure,
mechanism of action, targets, and sources. (p.196-197)
Exotoxin- secreted molecules that can kill, damage, or alter host cells. Endotoxin-see above
details.
Couldn't find the
H.) Describe 3 mechanisms by which exotoxins work and provide examples of bacterial diseases that are
caused by each of them. (p. 198-199)
NO IDEA
I.) Explain how bacteria can circumvent destruction by the host immune system in order to effectively
colonize humans and produce disease. (p. 198-201; Box 19-3)
The capsule is anti-complementary, and this helps retard complement deposition. If the complement is deposited, it
can be deposited so far beneath the capsule that the phagocytes can't ever see it. M-protein is a molecule from Group
A streptococci is anti-complementary and antigenically unique. On pneumoncocci, there are proteins on the surface
that are anti-complementary. Some pathogens, such as gonorrhea, can randomly alter its structure during an
infection.
J.) Describe 3 mechanisms by which certain bacteria can circumvent phagocytic killing after ingestion by host
phagocytes and provide an example of a bacterial species that utilizes each mechanism. (p. 200; Table 19-4)
Mycobacterium tuberculosis grows slowly so it doesn't stimulate much of an immune response, and its cell wall is
very greasy/oily which protects it from standard efforts of the lysosome. Listeria monocytogenes makes listeriolysin,
an enzyme that allows it to escape to the cytoplasm and go from cell to cell. Rickettsiae escapes by coming out into
the cytoplasm. Salmonella prevents lysosome-phagosome fusion.
K.) Explain the differences between active and passive immunity.


Active immunity is the development of antibodies in response to stimulation by an antigen.
Passive immunity. Once formed, those antibodies can be removed from the host and transferred into another
recipient where they provide immediate passive immunity
Active Immunity: is induced by exposure of the host to a foreign antigen – the host’s
immune system plays an active role in responding to the antigen
Active immunity confers specificity to the response
Active immunity also confers memory – the ability to respond against an
organism upon a secondary encounter
Example: getting sick or getting a vaccine
Passive Immunity: is conferred upon an individual by transferring serum or
lymphocytes from a specifically immunized individual – the recipient becomes
immune without having been exposed to or having responded to that particular
antigen
Passive immunization is useful for conferring resistance rapidly, without
having to wait for an active response to develop (~6-7 days)
Passive immunity confers specificity to the response
Passive immunity does not confer memory – the host would be susceptible to
a secondary encounter with the specific microbe
Passive humoral immunity is transferred by cell-free, antibody-containing
portions of the blood – serum or plasma whereas passive cell-mediated
immunity is transferred with cells (T cells)
Example: transfer of antibodies via breast milk
L.) Explain why a polyvalent polysaccharide-conjugate vaccine is used to immunize infants against invasive
pneumococcal disease whereas a polyvalent vaccine alone is used in adults at risk for invasive pneumococcal
disease. (p. 161-162)
Polysaccharides- which are common on pneumococci, hemophilus, and neisseria-these turn out to be very poorly
immunogenic in children less that two years of age. Have been conjugating the polysaccharides to some protein. This
elicits a response in children as if though they have been immunized to the protein. So by hooking it on to a protein it
confuses the immune system. Kids don't make Abs to polysaccharides.
M.) Describe the different mechanisms by which host resistance to infection by extracellular bacteria vs
intracellular bacteria occurs.
Extracellular bacteria replicate outside of cells and must avoid being killed by phagocytes or complement.
Ex: staphylococci
Intracellular bacteria replicate inside cells and must avoid being killed by phagocytosis and the antibacterial
properties of lysozomes. Ex: TB, Typhoid Fever
Chapter 3: Bacterial Genetics
A.) Describe differences between the bacterial and human genomes, including size, composition,
arrangement, presence of extrachromosomal elements, numbers of chromosomes. (p. 35)
*
1 circular DNA chromosome ranging in size from 1,000 to 8,000kb
*
Bacterial genome: the collection of all the genes present of the bacteria's chromosome or its extrachromosomal
genetic elements
Plasmid= Extrachromosomal genetic element
*
*
*
Genomes contain operons
Operons made up of genes
Promoters and operators control genes
B.) Explain 3 different mechanisms of transfer of genetic information between bacterial cells: transduction,
transformation, and conjugation.
Transduction – bacteriophage-mediated DNA transfer
Transformation – uptake of naked DNA
Conjugation – bacterial mating
Transduction: a bacteriophage (bacterial virus) is used to transfer genetic information inside bacterial cells
o Genes from an infected bacterium get mixed with bacteriophage genes during
infection
Steps in Transduction
1. Phage injects its DNA into host 1
2. Phage takes over host 1’s DNA replication enzymes to replicates itself
3 .Phage directs host cell to produce Dnase so it can chop the replicated DNA into
also chops up host 1’s DNA)
4. Newly replicated and chopped phage DNA gets packaged
5. By mistake some host 1 DNA also gets packaged
6. Mature phage leaves the infected cell carrying the host 1genes
7. Phage with host 1 DNA infects new host cell
appropriate sections (this
8. Genes from host 1 bacterium can get integrated to new host bacterium
9. Follows same integration pattern as transformation
10. Replicates and passes on to daughter cells forming colonies if cell remains transduced
Transformation: transfer of free or ‘naked’ DNA from the environment to a recipient cell
Competence: ability to take up DNA from the environment, can be natural vs. artificial
Natural: unknown reason why DNA is able to pass thru cell membrane barrier
Artificial: techniques like electroporation used to create tiny holes in the cell wall and
cytoplasmic membrane so that DNA can slip inside the cell
Steps in Transformation
1. dsDNA binds to competent cells
2. The donor DNA enters the cell as a single strand
other strand of donor DNA is degraded by nucleases at the cell surface
3. The donor DNA lines up complementary to the same region in the recipient DNA
4. Hydrogen bonds form between the donor and recipient DNA
5. Nuclease cleaves off the recipient DNA section where it is degraded by nucleases
6. The donor section replaces it in the dsDNA = integration (and technically a mutation)
7. After integration, dsDNA undergoes mismatch repair
Two possible outcomes:
o Remove the recipient DNA in the complementary strand to correct base-pairing (transformed)
o Remove the donor DNA in the original strand (untransformed)
8. Cells selected for transformed characteristic (i.e. streptomycin resistance)
Transformed cells form colonies
Untransformed cells die, and do not produce colonies
o Each cell now has mutation
Conjugation: transfer of plasmid from one cell to another cell via pilus
o Requires actual contact between donor and recipient cells (unlike transformation and
transduction)
o Donor cells contain a special plasmid F (fertility plasmid)
Donor cells designated F+
o Recipient cells have no plasmid
Recipient cells designated Fo
F plasmid encodes for the proteins for the sex pilus- F pilus
o F pilus acts as a conduit between the F+ (donor) and F- (recipient) cells
o Self- transmissible- contains all genes necessary for transfer from one cell to another
o F plasmids can give this genetic information to plasmids that are not selftransmissiable
originally
Steps in Conjugation
1. There must be contact between the donor and recipient cells!
o Sex pilus of donor cell recognizes a specific receptor site on the recipient cell
Acts like a grappling hook
2. Plasmid becomes mobilized or activated for transfer
o An enzyme produced by gene on plasmid cleaves the plasmid at its origin
3. A single strand of DNA from the plasmid in the F+ cell begins to enter the F- cell via the pilus conduit
o Leaves one strand of plasmid DNA in the F+ cell and it’s complementary strand in the F- cell
4. Single strand of plasmid DNA in the original F+ cell and the new F+ cell (formerly a Frecipient) replicates
Chapter 4: Antimicrobial Agents, Chemotherapy and Resistance
A.) Describe the differences between antibiotics, antiseptics, and disinfectants. (lecture outline)
Antibiotic: an agent that can be naturally occurring, partially or completely synthetic that selectively inhibits the
growth of microorganisms at low concentrations (ex. Penicillin)
Antiseptic: an agent used to inhibit or eliminate microbes on skin or other living tissue (ex. alcohol, iodine,
chlorhexidine)
Disinfectant: an agent used to destroy microbes on inanimate objects (ex. phenols, formaldehyde, chlorine bleach)
B.) Recognize the generic names of all antibiotics and groups discussed in the lectures and be able to classify
them according to mechanism of action. (lecture outline; Table 20.1)
Generic names of antibiotics and groups classified according to mech. of action
a) Cell wall synthesis: Cycloserine (non beta lactam), Glycopeptides (vancomysin) (non beta lactam), Bacitracin
(non beta lactam), Isoniazid (non beta lactams), Penicillins (beta lactam), monobactams (beta lactam), Carbapenems
(beta lactam), Cephalosporins (beta lactam),
b) RNA Elongation: Actinomycin
c) DNA gyrase: Nalidixic acid, Ciprofloxacin, Novobiocin (all quinolones)
d) DNA- directed RNA Polymerase: Rifampin, metronidazole
e) Protein Synthesis (50S inhibitors): Erythromycin (macrolides), Chloramphenicol, Streptogramins, Oxazolidinones
f) Protein Synthesis (30S): Aminoglycosides, Tetracyclines
g) Cytoplasmic Membrane Structure: Polymyxins, Daptomycin
C.) Describe the targets in the bacterial cell where antibiotics act to inhibit growth and how the various drugs
work at each site (e.g., cell wall, cell membrane, ribosome, DNA replication, etc.). (p. 204)
Targets in the bacteria cell where antibiotics act to inhibit growth and how the drugs work at each site
Cell Wall Synthesis:
 Cephalosporins: Slow diffusion due to bulk and ionic charge
 Cycloserine: Inhibits synthesis of D alanyl D ALA within cell called mucopeptide
 Glycopeptides (vancomysin): binds to D ALA D ALA moiety of precursor subunit blocking transpeptidation
 Bacitracin: complexes with lipid carrier that transports peptidoglycan precursors from cytoplasm to cell
membrane

Penicillin: Binds active site of transpeptidase enzyme that cross-links peptidoglycan by joining D-ALA and
GLY, Irreversibly inhibits transpeptidase- inhibits growth of the bacterial cell wall
 Isoniazid (INH): Inhibits fatty acid and lipid components of mycolic acid synthesis in mycobacteria
Bacterial Ribosome:
 Streptogramins bind 50S prevent peptide elongation and premature release from ribosome
 Oxazolidinones bind 50S and prevent formation of initiation complex
 Aminoglycosides bind 30S. inactivate initiation complex, misread mRNA genetic code and prematurely
release peptide
 Tetracyclines prevent attachment of rRNA AA to 30S
 Macrolides bind 50S and prevent release of deacylated tRNA preventing peptide elongation
 Chloramphenical and clindamycin bind 50S and prevent peptide bond formation.
DNA Replication:
 Rifampin: Binds DNA-dependent RNA polymerase and prevents mRNA transcription
 Metronidazole- intermediate metabolites damage DNA in anaerobes.
Cell Membrane:
 Polymyxins
 Daptomycin membrane depolarization
G.) Understand the difference between innate and acquired antibiotic resistance and give examples of each.
There are 2 types of microbiological resistance: innate and acquired.
Innate resistance is the natural resistance bacteria possess to some antimicrobials. For example, organisms may be
naturally impermeable to some antibiotics due to their cell structure. Organisms with innate resistance are often of
low virulence but, because they are resistant to so many agents, they persist in the environment.
An organism can also acquire resistance to an antimicrobial to which it was previously sensitive. This can be due to
chance mutation in the genetic material of the cell, or the acquisition of resistance genes from other drug resistant
cells.
H.) Understand the concept of clonal spread as it relates to transmission of antimicrobial resistant bacteria.
(lecture outline)
Clonal spread refers to the fact that bacteria can mutate and become resistant to certain drugs, and a person with a
drug-resistant bacteria can travel around the world spreading this drug resistant bacteria to other people.
I.) Understand differences between bactericidal and bacteriostatic drugs and how this relates to drug choice
in specific clinical settings. (lecture outline)
Bacteriocidal drugs kill the bacteria and are used mainly only in life threatening conditions such as endocarditis and
meningitis, or in an immunosuppressed host.. Bacteriostatic drugs only stop the bacteria from dividing, they don’t
kill the bacteria. In most infections, bacteriostatic drugs are used because preventing the bacteria from dividing is
generally enough to stop the infection. Caution must be taken when prescribing combination therapy because a drug
such as penicillin, which can act only when the bacteria is dividing, used in combination with tetracycline, which is
bacteriostatic(stops the bacteria from dividing), would therefore have antagonistic effects on each other.
J.) Explain the rationale for antibiotic prophylaxis and combination therapy. (lecture outline)
The rationale for antibiotic prophylaxis – mainly preventative measures such as:
-preventing endocarditis in susceptible persons undergoing dental work
-contacts of meningococcal meningitis
-contacts of skin test converters of TB
-opportunistic infections in HIV
-recurrent UTI’s
-prevention of post-surgical infections
-The rationale for combination therapy – advantages such as:
-providing a broader spectrum for infection of unclear etiology
-polymicrobial infections
-prevent emergence of resistance (Mycobacterium)
-infections difficult to eradicate (immunosuppressed host)
-synergy (drugs enhancing the actions of other drugs)
K) Know major characteristics of bacteria that favor development of drug resistance (lecture outline).
 Intrinsic resistance to some drugs
 Ability to exchange genetic information
 Ability to survive adverse environmental conditions
 Easily colonize, infect, and transmit
 Reservoirs in body
L.) Explain the reasons why drug resistance in bacteria is increasing in the hospital and community and its
consequences to the health care system.
Bacteria can persist in hospitals, where there is a high selection pressure (hospitals use a lot of antibiotics so
multiply-resistant bacteria are naturally selected), and can lead to difficult to treat nosocomial infections Species
currently causing treatment difficulties include Pseudomonas aeruginosa & Acinetobacter baumannii, although
these problems are also partly due to acquired resistance.
Chapter 5 Anaerobic Bacteria
A.) Describe 2 enzymes that are lacking in strict anaerobes and why the lack of these enzymes renders oxygen
toxic to them. (lecture outline)
Superoxide dismutase and catalase, most cytochrome systems are absent. These enzymes detoxify metabolic
byproducts in the presence of O2.
O2 → Metabolism → products toxic for enzymes → no detoxifying pathway → bacterial death
B.) Describe the anatomic sites where anaerobes are normal flora. (p. 83-87)
Oral cavity, Colon, and Genital Tract
C.) Learn clinical characteristics, responsible orgs, and mechanisms of disease for these conditions caused as
described in the lecture and assigned reading:
Periodontitis – no answer given
Brain Abscess: mostly Gram +, rods, cocci, and few gram negative; Bacteriodes, Prevotella, Porphyromona
Pulmonary Abcess: caused by Actinomyces, causes extensive soft tissue damage; cross tissue plane and
involve multiple organs; diagnose by sulfur granules coming out of sinus tracts
Vincent’s Angina: synergistic infection, gums recede from teeth, onvergrowth of Fusobacterium,
Bacteriodes, and Treponeme
Gas Gangrene: C. perfringens causing destruction of WBCs, platelets, and other host celss, H2 & CO2,
phosphlipase C (alpha toxin); diagnose by spore stain and has empty spots inside the cell; Identify C.
perfingens by Nagler test, agar impregnated with antitoxin on one side and no antitoxin on the other
Lumpy Jaw: caused by Actinomycosis, because of poor oral hygiene; diagnose by filamentous branching
gram positive rod killed by O2
Tetanus: caused by Clostridia (gram + spore), get vaccination to prevent tetanospasmin; causes continuous
firing of motor neurons
Botulism: food-borne illness, caused by Clostridium botulinum causes blockage of contraction of muscle,
ingestion of preformed neurotoxin except for wound infection and infarct botulism
Pseudomembranous colitis: antibiotic associated diarrhea cuased by C. difficile; diagnose by double zone of
hemolysis, makes multiple toxins; resistant to mnay antibiotics so hard to kill
D.) Understand how laboratory diagnosis of the above anaerobic infections is achieved.
Answered above for most of the organisms. In general to diagnose anaerobic infections have foul smelling
discharge, located close to mucosal surface, gas in tissue, have abcess formations; gram stain helpful in
establishment of mixed infection or presence of clostridia in wounds. In culture, require complex medium
supplemented with hemin, vitamin K, and blood. Should include media containing antibiotics to suppress
facultative anaerobes. Incubation and work up peromed in CO2 in nitrogen/hydrogen mix
E.) Explain the differences between the following:
Aerobic bacteria: require oxygen as electron acceptor
Microaeirophilic: require O2 in reduced quality
Capnophilic: require CO2
Facultative: grow either with or without O2
Anaerobic: both obligate (killed by any level of O2) and aerotolerant (grow in small amount of oxygen)
Chapter 6: Important concepts related to specific groups of bacteria
A.) Gram—positivie coccie and bacilli
1. How do Protein A and the P-V leucocidin aid Staphyloccoccus aureus in causing disease? (lecture outline)
Protein A aids Staph aureus in causing disease by binding the Fc region of IgG. This inhibits the Fab region from
binding and opsonizing the bacteria. Leucocidin aids Sa in producing disease by lysing white blood cells and
therefore releasing lysosomal enzymes which causes nearby tissue damage.
2. What advantage does the presence of coagulase confer on Staphylococcus aureus? (lecture outline)
Staph. aureus, an extracellular pathogen, has to escape phagocytosis in order to be successful and spread and
multiply in the host. Staph. aureus contains coagulase (clots plasma by converting fibrinogen into fibrin) around
itself in the body and human tissues. This helps Staph. aureus protect itself from WBCs, antibody, and complement,
and other things that would try to engulf it and kill it. That’s the way it has of getting around the host immune
response.
Catalase is used by organisms such as Staphlococci to break down hydrogen peroxide into water and oxygen.
2 H2O2 → 2 H20 + O2
A serologic classification of bacteria of Β hemolytic streptococci into groups (A-T) based on their specific
carbohydrate antigen. All strains pathogenic to man belong in group A.
3. Explain the concept of a “superantigen” and how the staphylococcal toxic shock toxin initiaties tissue
damage in the Toxic Shock Syndrome. (p. 224)
A key component of Toxic Shock Syndrome (TSS) is the ability of several toxins of Staphylococcus aureus to act as
“superantigens.” These superantigens bind simultaneously and non-specifically to both TCR’s and MHC Class II
receptors at the same time.
Superantigen binding of TCRs and MHC class IIs on antigen presenting cells (APCs) activates the T lymphocyte.
Because superantigens bind nonspecifically, polyclonal populations of CD4 T cells are activated and begin
proliferation. The large numbers of effector CD4 T cells resulting from this nonspecific proliferation begin
stimulating monocytes to secrete several cytokines, including tumor necrosis factor (TNF)  and interleukin (IL) 1.
The nonspecific, high volume superantigen stimulation of T cells results in systemic secretion of these cytokines
(instead of the localized secretion that normally occurs during infection), which causes much of the tissue damage
and morbidity associated with TSS.
5. Compare and contrast the virulence factors and diseases caused by Staphylococcus aureus versus the
coagulase-negative staphylococci. (p. 228-233)
virulence of s. aureus: B lactamase, pyogenic, capsule, protein A
diseases of s. aureus: impetigo, toxic shock, scalded skin, food poison
virulence of coagulase (-) staph: opportunistic infections, polysaccharide "slime"
6. Acute glomerulonephritis and rheumatic heart disease are both autoimmune sequelae of Group A
streptocolccal infections. Explain their similarities and differences with respect to immune mechanisms
involved and clinical manifestations. (p. 244-245)
A common bacteria that is the cause of strep throat, scarlet fever, impetigo, cellulitis-erysipelas, rheumatic fever,
acute glomerular nephritis, endocarditis, and group A streptococcal necrotizing fasciitis. The prototype is
Streptococcus pyogenes. DPNase kills white blood cells.
9. Describe the pathogenesis of streptococcal necrotizing fascitis. (p. 244)
Necrotizing fasciitis is a progressive, rapidly spreading, inflammatory infection located in the deep fascia, with
secondary necrosis of the subcutaneous tissues. Because of the presence of gas-forming organisms, subcutaneous air
is classically described in necrotizing fasciitis. This may be seen only on radiographs or not at all. The speed of
spread is directly proportional to the thickness of the subcutaneous layer. Necrotizing fasciitis moves along the deep
fascial plane.
11. Explain why the Enterococcus is especially well-suited to be a nosocomial pathogen. (p. 260-262)
Enterococcus is part of the normal flora of the GI tract and is particularly resistant to a broad range of antibiotics.
Long-term hospital patients treated with antibiotics are susceptible to opportunistic infections by Enterococcus.
12. What unique feature of Bacillus anthracis help to differentiate from other Bacillus species in the clinical
laboratory? (p. 267)
The capsule of B. anthracis is composed of a poly-D-glutamic acid. The capsule is a major determinant of virulence
in anthrax. The capsule is not synthesized by the closest relatives of B. anthracis, i.e., B. cereus and B. thuringiensis,
and this criterion can be used to distinguish the species.
13. Discuss the features of Bacillus anthracis that make it such a logical organism for bioterrorism attacks. (p.
265-269)
Like other organisms from the genus Bacillus, B. anthracis produces incredibly resilient endospores which are
capable of surviving for many years in “hostile” conditions. The spores can be transported and spread easily and will
be become activated upon inhalation by a human host.
14. Describe the epidemiology and pathogenesis of meningitis due to Listeria monocytogenes, E. coli, Strep
agalactiae, Neisseria meningitidis, and Haemophilus influenza and emphasize the differences and similarities
among them.
Listeria monocytogenes:
- causes meningitis in newborns, the immunosuppressed, pregnant women, and cancer patients
- acquired by ingestion of contaminated food, it may be asymptomatic in healthy adults
- the organism can cross the placenta and cause spontaneous abortion and stillbirth
- organism can grow in refrigerator and infection can come from dairy products
E. coli:
- neonatal meninigitis
- approximately 75% of E. coli strains possess the K1 capsular antigen
- this serogroup is also commonly present in the GI tracts of pregnant womand newborn infants
Neisseria meningitidis:
- purulent inflammation of meninges associated with headache, meningeal signs, and fever
- high mortality rate unless promptly treated with effective antibiotics
- approx. 0.6 cases per 100,000 population reported in 2004
- very young children may have only nonspecific signs such as fever or vomiting
- incidence of neurologic sequelae is low, with hearing deficits and arthritis most commonly reported
Haemophilus influenzae:
- used to be a common cause of pediatric meningitis, but not anymore due to wide use of conjugated
vaccines
- dz in non-immunized patients results from bacteremic spread of the organisms from the nasopharynx and
cannot be differentiated clinically from other causes of bacterial meningitis
- initial presentation is 1-to-3 day history of mild upper respiratory dz after which the typical signs of
meningitis appear
Streptococcus agalactiae:
- carried in GI tract of pregnant women as normal flora
- if acquired by baby at delivery, can cause severe pheumonia and severe meningitis
- most common cause of septicemia and meningitis in newborns
- approximately 60% of infants born to colonized mothers become colonized with their mother’s organisms
- risk factors for neonatal colonization—premature delivery, prolonged membrane rupture, intrapartum
fever
- serotypes most commonly assoc. with neonatal dz are Ia, III, and V.
- serotypes Ia and V are the most common in adult dz
-
colonization of neonate can occur in utero, at birth, or during the first few months of life
risk of adult dz greater in pregnant women than in men and non-pregnant women
B.) Gram-negative bacilli
1. Compare and contrast diarrheal diseases caused by Salmonella, Shigella, Campylobacter, Staphylococcus
aureus, Bacillus cereus, Escherichia coli, Yersinia, and Vibrio species with respect to epidemiology, modes of
treansmission, mechanism (infection vs. intoxication), and invasiveness.
Salmonella- (grows black on XLD agar- shows H2S). Salmonella really invades our cells. It causes a profuse
diarrhea. Salmonella causes a few million cases of food borne illness each year in the US. Most of it is not so severe
that it is identified. There are 2400 serotypes, but there are only 2 of those that cause 40-50% of all salmonella
outbreaks- enterica Typhimurium, and enterica enteritidis are the most common. You have vomiting and perfuse
diarrhea for 8-48 hours after you start developing symptoms and this then resolves in 7-10 days. The source of
salmonella is usually our meat animals or people. The way to prevent this organism is to cook it. (Don’t cut chicken
and salad on the same cutting board or use the same knife!)
Shigella- causes bacillary dysentery, a high volume, watery diarrhea. It invades the colon epithelium and can be
treated with Ampicillin or Bactrim. Humans are the only normal host. 200 organisms is an infectious dose, which is
a very low amount in comparison to most bacterial infections. It can be transmitted by the 4Fs: feces, fingers, food,
flies, and water. It can be controlled by washing your hands and other sanitary measures in society. Common sites
of this disease are nursing homes, day cares, wars, and cruise ships. It causes 15% of the pediatric diarrhea in the
US.
Campylobacter- It causes a bloody diarrhea. It is more common and less severe than Salmonella. It is carried by
food animals and pets. Most human infections are from contaminated food and water sources. Treatment consists of
rehydration. You can prevent this organism through cleanliness and cooking your food.
Staph aureus- causes watery diarrhea. It is a preformed enterotoxin that is heat stable. It is in foods such as ham,
cream-filled cakes, and potato salad. It’s toxins infect kittens and non-human primates. The effects of this organism
usually last less than 24 hours.
Bacillus cereus- is a preformed enterotoxin that is heat stable. The diarrhea produced is profuse and watery. Several
of the symptoms are much like staph. The difference is that the foods are typically a starchy food instead of a protein
rich food. Those spores will germinate after it is cooked, grow in the rice, and then refry the rice and kill the
organisms again, but the toxin will stay around and cause disease. A large number of spores germinate in vivo.
E. coli- causes secretory diarrhea or watery diarrhea. Features of this diarrhea include stimulation of net intestinal
secretion, no morphological damage, and no impairment of Na- dependent solute absorption. Species of E.coli are
responsible for traveler’s diarrhea and infantile diarrhea. Eneterovasive E. coli (EIEC) causes bacillary disentry.
The most common outbreak strain in food borne illness is EHEC, or Enterohemorrhagic E. coli. The diarrhea present
is usually mild.
Yersinia- is an enterobacteriaceae but it is not food borne. It causes a systemic disease much like salmonella typhi.
It causes the bubonic and pneumonic plague and is contracted by flea infected bites. It causes septicemia and often
death. Pneumonic plague is passed from person to person coughing up the organisms. The plague in 14th century
Europe killed 25 million people and was often of the pneumonic type. Bubonic plague can result in 75% mortality
with a few days; the pneumonic form can result in greater than 90% mortality within 24 hours. Control of rat
populations concurrent with elimination of their fleas prevents spread of the plague to humans. Decontamination of
water and milk prevents gastroenteritis. Treatment of the plague must be rapid and aggressive. Y. pestis is generally
susceptible to streptomycin and chloramphenicol but concomitant therapy is sometimes recommended. Treatment of
Y. enterocolitica infections usually involves the use of ampicillin or tetracycline.
Vibrio- Toxins produced in vivo. It produces a profuse watery diarrhea. It is spread by contaminated water.
Stomach acid offers protection. An infectious dose is 107 CFU. It has a 1-5 day incubation period. Rehydration by
fluids is sufficient treatment.
2. Describe 4 characteristics that can be used to define the family Enterobacteriaceae. (p. 323)
A. Gram negative Rods
B. All have the enterobacterial common antigen
C. Non spore forming, facultative anaerobes that can grow on a variety of nonselective (blood agar) and
selective media (MacConkey).
D. Ferment glucose; reduce nitrate, catalase positive and oxidase negative.
3. Which property of Helicobacter pylori enables it to reside in the stomach at pH of 2 and how does this
relate to the ability of the organism to induce an inflammatory reaction. (p. 352)
Bacterial acid inhibitory protein blocks acid production and bacterial urease activity produces ammonia
which neutralizes gastric acid. Then the bacterium passes through the gastric mucus and adheres to the epithelial
cells which are then damaged by urease by products. Epithelial damage , urease, and LPS, stimulate the
inflammatory reaction.
4. Which virulence factor of Pseudomonas aerugionsa contributes to its pathogenesis in burn patients? (p.
359)
Exotoxin A: disrupts protein synthesis by blocking peptide chain elongation in eukaryotes.
5. Discuss the relative organism load needed to cause diarrheal disease due to Escherichia coli, Shilgella spp.,
Salmonella spp. And Vibrio cholera. (lecture outline)
E. coli—The specific amount of E. coli needed to cause diarrhea was not listed in the hand out. But, from lab and
other sources, the number has to be relatively high since it is part of our normal flora. I think the number is
somewhere around 10^6.
Shigella spp.—The amount needed to cause an infection is only 200 organisms. However, there are
approximately 10^8 colony forming units/gram (CFU/g) in infected stool samples. Obviously this puppy could
spread pretty quickly.
Salmonella spp.—Again, the specific amount is not in the handout. I could not find any solid info on the net
to even make a guess at it. Sorry I could not be of more help on this one.
Vibrio cholerae—The amount needed to cause infection is 10^7 CFU. This high number is due to the fact
that the stomach acid will kill most of the bugs on the way in. The load has to be very high in order to get past the
stomach and produce diarrhea.
6. Explain the difference between osmotic and secretory diarrhea. (lecture outline)
Osmotic diarrhea occurs when unabsorbable, water-soluble solutes remain in the bowel, where they retain water.
Osmotic diarrhea occurs with sugar intolerance, including lactose intolerance caused by lactase deficiency, and with
the use of poorly absorbed salts (Mg sulfate, Na phosphates) as laxatives or antacids. Ingestion of large amounts of
the hexitols (eg, sorbitol, mannitol), which are used as sugar substitutes, causes osmotic diarrhea as a result of their
slow absorption and stimulation of rapid small-bowel motility.
Secretory diarrhea occurs when the small and large bowel secrete more electrolytes (decrease Na absorption, increase
Cl secretion) and water than they absorb. Mucosal cAMP increases the effects of the Na and Cl mentioned above.
Secretagogues include bacterial toxins (eg, in cholera), enteropathogenic viruses, bile acids, unabsorbed dietary fat,
some drugs, castor oil, prostaglandins, and peptide hormones. Fasting does not stop diarrhea (quickly).
7. What is the hemolytic-Uremic Syndrome and how is it related to bacterial infection? (lecture outline)
Hemolytic uremic syndrome is one of the most common causes of sudden, short-term kidney failure in children.
Most cases of HUS occur after an infection of the digestive system by Escherichia coli bacterium, which is found in
contaminated foods like meat, dairy products, and juice. Some people have contracted HUS after swimming in pools
or lakes contaminated with feces.
C.) Fastidious bacteria
1. Describe the process by which Mycoplasma pneumoniae attaches to the respiratory epithelium and
produces pneumonia. (lecture outline)
Mycoplasma pneumoniae expresses P1 and other related proteins on its surface which allow attachment to respiratory
epithelia. Once attached, the bacteria can produce hydrogen peroxide which can actually halt the ciliary activity of
the respiratory epithelium. This, in turn, results in inflammatory responses in which cytokine cascades are activated.
The virulence attributed to this bacteria is correlated to the lipid-associated membrane proteins that are exposed on
the cell surface. The expression of specialized polar tip organelles for mediating attachment to host cells is a
coordinate interaction between designated adhesins, interactive proteins, and adherence-accessory proteins. By
concentrating adhesins at the tip of this structure, mycoplasma are able to colonize mucous membranes and
eukaryotic cell surfaces. M. pneumoniae possesses only one full-length copy of the adhesin genes in a single operon,
but has available to its defenses, multiple copies of very similar adhesins. These copies can be utilized through
homologous recombination to evade the host immune response. Research has shown that pathogenic mycoplasma
choreograph the rearrangement of DNA using the multiple copies of adhesin gene sequences to supercede the natural
disadvantages to having a small genome. This "gene shuffling" also allows for a high rate of reinfection of patients.
M. pneumoniae does not have to evade the host immune response, necessarily, because it is able to create an
"antiself" response. By serving as B-cell and T-cell mitogens, mycoplasmas could be able to activate antiself T cells
or polyclonal B cells.
2. Explain the differences in clinical manifestations, epidemiology, natural history, and pathogenesis of
pneumonia caused by the following organisms.
(C2page9)
BUG
Streptococcus
pneumonia
Staphylococcus
aureus
Klebsiella
pheumoniae
Clinical Manifestations
-onset abrupt
-severe shaking chill and sustained high
fever
-often preceeded by symptoms of viral resp.
tract infection 1 to 3 days before onset
-chest pain, blood-tinged sputum
-often lobar pheumonia, associated with
aspiration
-radiograph reveals patchy infiltrates with
abscesses
Clinical presentation NOT unique
-thick non-purulent blood sputum
Epidemiology
Natural History
Pathogenes
-common inhabitant of throat and
nasopharynx in healthy people
-common cause of bacterial
pneumonia
-500,000 cases in US annually
-endogenous oral org
aspirated into the low
-seen in very young and elderly and
patients with underlying or recent
pulmonary disease
-severe form of necrotizing
pneumonia with septic shock and
high mortality
-25-50% mortality
-community-acquired primary lobar
pneumonia
-consolidation and ab
formation in lungs
-can occur by aspirat
from the hematogeno
of the organism from
sites
-necrosis and abscess
Predisposing factors:
-hospitalization
-respirator
-increased age
-alcoholism
-Diabetes Mellitus
-atypical interstitial “walking” pheumonia
-clinically similar to the other pheumonias
-all ages affected, but more common
in younger persons
-reinfection common—no protective
immunity
Chlamydophila
pheumoniae
-frequently asymptomatic
-acute lower respiratory illness, pharyngitis,
sinusitis
-persistent cough and malaise, but normally
does NOT require hospitalization
-similar to mycoplasma
Most infectious occur in adults
Diagnosis difficult
Infections transmitted person-toperson by respiratory secretions
-NO animal reservoir identified
Haemophilus
influenzae
-inflammation and consolidation of the
lungs observed primarily in the elderly with
underlying chronic pulmonary dz
-commonly colonize pts. Who have
chronic pulmonary dz
-frequently assoc. with exacerbation
of bronchitis and frank pneumonia
-respiratory aerosol dissemination
-2-10 day incubation period
-requires antibiotic therapy
-15-20% mortality rate; higher if diagnosis
delayed
-multilobar
-Legionnaire’s Dz
-severe pneumonia
-epidemic, sporacid
-no person-to-person spread
-epidemic dz in late summer and
autumn
-there is normally an underlying
pulmonary dz
Mycoplasma
pheumonia
Haemophilus
influenzae
(cont’d)
Legionella
pheumophila
1980s labs testing antibody on
people with respiratory
infection were getting lots of
positive tests for Chlamydia
psittaci, but they hadn’t been
around birds, so this is when
they realized it was another
type of Chlamydia (now called
Chlamydophila pheumoniae)
American Legion Conference,
disease spread through AC
ducts and water supply
(possibly) and infected many
Legionnaires, hence the name.
-cytadherence: P1 an
proteins
-altered macromolec
synthesis
-induction of inflamm
-human pathogen
???
-typically caused by
nontypeable strains
-endogenous infectio
normal flora
-antiphagocytic caps
-endotoxin damages
epithelium leading to
spread
-IgA protease
-organism can surviv
environments for a lo
at high temps. And in
of disefectant like ch
-organism proliferate
reservoirs during wa
3. Discuss the impact of the Hib vaccine on epidemiology and spectrum of disease caused by Haemophilus
influenzae in adults and children. (lecture outline)
Wherever thorough studies have been performed, Haemophilus influenzae type b (Hib) has been shown to be an
important cause of childhood meningitis and a major cause of bacterial pneumonia in children. The most important
manifestations of Hib disease, namely pneumonia and meningitis, are seen mainly in children under five years of age,
particularly infants. Currently, several different Hib vaccines, all conjugate vaccines, are on the market. These
vaccines have shown protective efficacy in early infancy. Hib vaccines have proved to be highly efficacious and
virtually free from serious side-effects. Because these vaccines significantly reduce nasopharyngeal carriage, a herd
effect is achieved through Hib vaccination.
4. Explain why H. Influenzae must be supplied with X and V factors in order to grow on trypticase soy agar.
(lecture outline; p. 372)
H. Influenzae requires X and V factors to grow and these factors are not present in trypticase soy agar as they are in
chocolate agar.
5. Explain why Bordetella pertussis infections have increased in adolescents and adults in the United States
over recent years. (lecture outline)
Usually children get boosters against it before they enter school. With time, immunity declines, so adolescents and
adults become more susceptible and can spread the disease. Also, there is no vaccine licensed for people greather
than 7 years old, so this makes the spread more likely among adults.
6. Compare and contrast the diseases of diphtheria and pertussis, including their epidemiology, pathogenesis,
and prevention. (lecture outline)
Diphtheria—
- spread by droplets
- humans only known reservoir
- 2-6 day incubation period
- Nasopharynx and larynx mainly affected
- Pseudomembrane firmly adheres to respiratory tissue and is difficult to dislogdge without making
underlying tissue bleed
- Course of disease is about 1 week
- 5 cases per year in US since 1980
- Exotoxin activity causes damage
Pertussis—
- disease spread person-to-person by infectious aerosols
- human reservoir
- 5-21 day incubation—very contagious
- Children <1 year old and non-vaccinated people at greatest risk for infection
- Course of disease is >4 weeks
- 12,000 cases in US in 2004
- Acellular vaccines containing inactivated pertussis toxin highly effective (administered in 5 doses)
7. Explain why the initial outbreak of legionellosis in 1976 was so difficult to characterize initially. (lecture
outline)
The Legionella bacterium was first identified in the summer of 1976 during the 58th annual convention of the
American Legion, which was held at the Bellevue-Stratford Hotel in Philadelphia. The presentation of affected
persons ranged from mild flulike symptoms to multisystem organ failure. Of the 182 people infected, 29 died. A
bacterium that would later be named L pneumophila was isolated from different organ tissues of guinea pigs
inoculated with lung tissue samples from 4 fatal cases. Several problems were encountered when researchers first
tried to classify this outbreak. First, the bacteria are difficult to culture. It would not grow on standard media, and a
buffered charcoal yeast extract agar was developed to culture it. Also, characterization was hindered due to the lack
of person to person contraction. The source was not known since none of the affected persons had symptoms prior to
arriving at the hotel. It was later discovered that Legionella is water-born and was actually aerosolized in the airconditioning system.
8. Describe 5 methods for laboratory diagnosis of legionellosis and discuss the advantages and disadvantages
of each. (p. 394)
Legionella pneumophila fastidious
 catalase neg
 gram neg bacillus
 nonfermentive, facultively intracellular
 no person to person transmission
 acute purulent pneumonia
5 methods for laboratory diagnosis of legionellosis, advantages and disadvantages
1. silver staining of histopathology specimens-direct method
2. direct fluorescent antibody (poor sensitivity)-direct method cannot be performed on speutum, delayed results
waiting for growth of bacteria, detects all serogroups
3. urine polysaccharide antigen- detected 1-3 days after diagnosis and persists up to a year, non-invasive, best for
serogroup 1 detection
4. serology IFA-paired sera required, not good for detection in acute phase
5. PCR-amplify results, blood PCR stays positive longer than speutum PCR, non-invasive
9. What are the consequences of having the smallest genome of any know free-living human pathogen on
laboratory diagnosis of Mycoplasma genitalium infections? (lecture outline)
The small genome of M. genitalium requires an enriched medium (SP4 agar) and a lot of time in order to grow in a
laboratory. As a result, PCR and serology (for IgM) are used more commonly to identify the presence of the bug.
D.) Spirochetes and Rickettsiae
1. Explain the differences between obligate intracellular bacteria and facultative intracellular bacteria and
give examples of each. (lecture outline)
Obligate intracellular bacteria are restricted to an intracellular way of life. They frequently cause chronic disease.
Example: Rickettsiae and Chlamydiae
Facultative intracellular bacteria have developed the ability to survive and grow within professional phagocytes
frequently giving rise to chronic and/or chronic disease. Example: tuberculosis, Salmonella typhimurium, Listeria
monocytogenes.
2. Describe the life cycle of Borrelia burgdorferi, the agent of Lyme disease. (lecture outline)
For Lyme disease to exist in an area, at least three closely interrelated elements must be present in nature: the Lyme
disease bacteria, Borrelia burgdorferi, ticks that can transmit them, and mammals (such as mice and deer) to provide
food for the ticks in their various life stages. The tick life cycle consists of three distinctive stages: larvae, nymphs,
and adults. A blood meal is required for ticks to molt from the larvae stage to the nymph stage and from the nymph
stage to the adult stage. The tick larvae and nymphs typically become infected with Borrelia burgdorferi when they
feed on infected small animals, particularly the white-footed mouse. The bacteria remain in the tick as it changes
from larva to nymph or from nymph to adult. Infected nymphs and adult ticks then bite and transmit Borrelia
burgdorferi to other small rodents, other animals, and humans, all in the course of their normal feeding behavior.
Adult ticks preferentially feed on the white-tailed deer, which thereby becomes an important reservoir in regions of
infestation. The tick life cycle takes two years to complete.
3. Does Lyme Disease occur endemically in Alabama? Explain the reason for your answer. (lecture outline)
Lyme disease does not occur endemically in Alabama. This is because Lyme disease is transmitted by Ixodes ticks
and these are not native to Alabama.
4. Explain why laboratory diagnosis of Lyme Disease is complex and difficult. (p. 436 in text)
Diagnosis is complicated by the varied manifestations of disease caused by B. burgdorferi and other Borrelia
species, as well as the lack of reliable diagnostic tests. B. burgdorferi are present in very small amounts in most
clinical specimens from tissues and body fluids of patients with Lyme Disease, so microscopic examination and even
nuceic acid amplification have low sensitivity. Cultures are rarely done because media is NOT readily available and
organisms grow slowly on them anyway.
5. What is the difference between a “non treponemal test” and a “treponemal test” and how can each be used
most effectively in diagnosis of syphilis. (p. 431-432)
Non-treponemal tests (eg: VDRL or RPR) are biologically nonspecific and they measure IgG and IgM
antibodies developed against lipids released from damaged cells during the early stage of disease and present on the
cell surface of treponemes. Treponemal tests are specific antibody tests used to confirm a positive reaction with the
VDRL or RPR test.
Non treponemal tests: Positive reactions with these tests develop late during the first phase of disease; the
findings are negative in many patients who initially have chancres. However, serological results are positive within 3
months in all patients and stay positive in untreated patients with secondary syphilis. Successful treatment of
primary or secondary syphilis leads to reduced titers measured in the nontreponemal tests and thus can be used to
monitor the effectiveness of therapy.
Treponemal tests: results of these tests usually remain positive for the life of the person who has syphilis.
These tests are influenced less by therapy than the nontreponemal tests
6. Explain how the life cycle of Rickettsia rickettsii is responsible for clinical manifestations of Rocky
Mountain Spotted Fever. (lecture outline)
Rickettsia rickettsii are carried in the fluids of arthropods like ticks. When the tick bites you, the bacteria will invade
the endothelium and the cells of the small vessels causing vasculitis and the spots charactierisitic of Rocky Mountain
Spotted Fever. The cells swell and become necrotic and thrombus of the vessals can happen making more spots. If
untreated can progress to DIC and bleeding in the brain.
7. Describe the procedures that are necessary in order to cultivate reickettsiae in vitro and explain whether or
not this is a worthwhile method for diagnosis of rickettsial diseases. (p. 452)
Rickettsia require cell culture in order to grow in labs, won’t grow on plates because they are obligate intracellular
bacteria. This is an uncommon practice because Rickettsia can spread and infect when you’re working with them.
Usually diagnose by detecting presence of Ab instead of cell culture.
8. Describe 3 different human diseases for which ticks are a vector.
Rocky Mt. Spotted Fever: Caused by Rickettsia rickettsii
Monocystic ehrlichiosis: caused by Ehrlichia chafeensis
Granulocytic ehrlichiosis: caused by Anaplasma phagocytophilum
Lyme Disease: caused by Borrelia burgdorferi
F.) Neisseriae and Chlamydiae
1. Describe the unique life cycle of Chlamydiae and the roles of the elementary body and reticulate body in
infectivity. (lecture outline)
FUN FACT: Chlamydia is an obligate intracellular organism (it doesn’t have the ability to make its own ATP)
Life Cycle: infectious elementary body taken up into cytoplasm of host cell>>>grows and becomes metabolically
active inside of phagosome (transformational change to reticulate body)>>>replication and release of more
elementary bodies by the reticulate bodies after about 3 days.
Elementary body: the small infective particle
Reticulate body: replicating form of Chlamydia trachomatis; non-infectious, but makes more elementary bodies
which are infectious
2. Give an explanation for the failure to develop an effective vaccine against Neisseria gonorrhoeae. (p. 319)
The pili of Neisseria gonorrhoeae acts as an antigen however there is antigenic diversity of different gonoccocal
strains. The antigenic diversity of the pilin proteins explains the ineffectiveness of vaccines.
3. Describe the diagnostic method of choice for chlamydial urogenital infections and explain why it is
preferred over other methods. (p. 469)
The NAAT (nucleic acid amplification tests) is the test of choice for the laboratory diagnosis of genital C.
trachomatis. Voided urine from a patient with urethritis can be used and it must be carefully monitored for presence
of inhibitors to the amplification reaction and to prevent cross contamination of specimens. NAATs are highly
sensitive (90-98%) and if properly monitored are very specific.
G.) Mycobacteria
1. How do the methods of lab diagnosis for mycobacteria differ from those used for the common gram-positive
cocci? (p. 305-308)
Must use the acid fast stain using acid alcohol instead of acetone alcohol which is used in the gram stain. Mycolic
acids, waxes, obligate aerobes, slow growing can use the fluorescent stain
2. Give a logical argument why the BCG vaccine is not used routinely in the United States. (p. 301)
BCG is not used in the US because it makes a positive PPD test, so we give skin test to figure out who has the
disease and treat only those infected. Also it prevents childhood disease, but not latent disease
3. Is tuberculosis increasing or decreasing in the U. S.? Explain your answer. (p. 301; lecture outline)
Decreasing b/c of genetic pool and standard of living
4. Explain why persons with HIV/AIDS are especially prone to develop disease due to mycobacteria. (p. 301)
Because of low immune system; However, the vaccine they are given prevents the primary disease of TB, so stops
the infection. But they are a carrier and can transmit it to other people.
5.A 30 year-old dentist in apparent good health who is expose to a patient with tuberculosis develops a positive
PPD skin test. Expalin the most appropriate course of action that should be taken.
Get chest x-ray or AFB culture to determine what drugs to treat it with. Can take isoniazids if positive PPD to
decrease the chance of acquiring it
6.Compare and contrast the pathogenesis and clinical manifestations of tuberculoid and lepromatous leprosy.
(lecture outline)
Tuberculoid leprosy: immune response to organism, grow in skin and nerves, lose feeling and wind up losing body
part because can’t feel that body part and don’t take care of themselves because it becomes infected and they don’t
know it. It doesn’t grow on artificial media, grows within mice without immune system and in armadillos. Must
AFB stain sucous to diagnose. Don’t have many organisms in skin. Treatment: dapsone and rifampicin at least one
year; Prevention: isolation of lepromatous case, vaccines under development.
Lepromatous leprosy: don’t develop immune response to organism; lots and lots of organisms because don’t have
immune response to organism and grow in high numbers in macrophages; bacteremia with localization in nerves and
skin; less loss of nerve function; leonine facies; other organs involved are testes, spleen, and liver.
Important Bacteria
Gram-positive cocci
Staphylococcus:
Staphylococcus aureus
a.) Gram positive
b.) Cocci in clusters
c.) Lives in anterior nares in healthy people, skin and mucous membranes, hospital environment
d.) Reduces phagocyte killing (Catalase), clots plasma (coagulase), destroys connective tissue (Hyaluronidase),
destroys beta lactam drugs (Beta lactamase), lyse WBCs and release lysosomal enzymes (cytotoxins and
leukocidins), interrupts intercellular skin junctions—“Scalded Skin Syndrome” (Exfoliatin), stimulates T
cells and endothelial damage and TSS (Toxic Shock Toxin)
e.) ??
f.) ??
g.) Aerobic culture, beta-hemolytic, Mannitol fermentation, Catalase positive, and Coagulase positive
h.) ??
i.) Incrfease in oscucillin-resistant organisms that are resistant to most other antibiotics except Vancomycin
(MRSA).
Staphylococcus epidermidis
a.) Gram +
b.) Coccus
c.) Resident skin flora, gut, upper respiratory tract
d.) Adhesion, capsule, slime, biofilm
e.) Common colonizer of skin but also is an important nosocomial pathogen; most common microorganism causing
infections after implantation of orthopedic devices, etc.
f.) Well adapted for causing shunt and catheter infections because they produce a polysaccharide slime that bonds
them to catheters and shunts and protects them from antibiotics and inflammatory cells; hydrophobic nature of the
organism’s cell surface facilitates its adherence to synthetic devices; many isolates are resistant to multiple
antibiotics
g.) Growth on blood agar
h.) Catalase +; coagulase negative; gamma hemolytic, susceptible to Novobicin
i.) Removal of catheters and prosthetic devices, antibiotics
Staphylococcus saprophyticus
a.) Gram +
b.) Coccus
c.) Pathogenic in bladder, urinary tract of sexually active young females
d.) Unknown
e.) Causative agent in 10-20% of UTIs in sexually active yound women. Little is known about its pathogenesis
f.) UTI in sexually active young women
g.) Growth on blood agar
h.) Gamma hemolysis, catalase +, coagulase negative, Novobicin resistant
i.) antibiotics
Streptococcus
Streptococcus Pyogenes
a.) Gram +
b.) Cell Morph: Cocci
c.) Reservoir: Humans- not in normal flora
d.) Virulence Factors: Hyaluronic Acid capsule that is antiphagocytic. Hyaluronidase can eat up the HA and
penetrate through the tissue, Beta hemolytic, M protein that is present on the pilus with teichoic acid and prevents
opsonization, binds fibrinogen and fibrin to form a dense coating to block it from complement. Streptokinase forms
the fibrin clots.
e.) Epidemiology: person to person, mainly in kids in the winter
f.) Disease type and spectrum: Scarlet fever, Erysipelas, Bacteriemias
Erythrogenic toxin- scarlet fever – toxin carried on bacteriophage
g.) Means of detection and diagnosis: culture, catalase -, Beta Hemolytic, Bacitricin Sensitive
h.) Unique features: None
i.) Ab to M protein but repeated infections can occur, Bacitracin sensitive, no vaccine.
Streptococcus agalactiae (Group B)- Hemolytic
A)
B)
C)
D)
E)
Gram stain rxn: gram +
Cell morphology: cocci in chains
Natural habitat or reservoir: GI tract (importantly in pregnant women as a part of their normal flora)
Major virulence factors: hyaluronic acid capsule (produces hyaluronidase)
Epidemiology: transmitted at time of delivery from mother to newborn infant; also seen in older debilitated
adults
F) Disease type & spectrum: causes severe pneumonia and meningitis
G) Means of detection or diagnosis: identified by hemolytic pattern; catalase neg. rxn; positive CAMP test
(streak Staph aureus on blood Agar and take strain of  strep and streak it perpendicular to it, group B strep
secrete a protein (CAMP factor) which enhances the hemolytic action of the staph -lysin on blood agar); can
also use Hippurate Hydrolysis to distinguish
H) Unique features: capnophile (like increased levels of CO2)
I) Prevention: pregnant women are screen for vaginal and rectal caries of group B strep
Streptococcus pneumoniae (Alpha Hemolytic)
J) Gram stain rxn: gram +
K) Cell morphology: cocci in PAIRS
L) Natural habitat or reservoir: respiratory tract
M) Major virulence factors: (all are listed, but some were not discussed in class)
hyaluronic acid capsule (produces hyaluronidase)
PspA (a surface protein that inhibits opsonization)
autolysin (releases cell components)
pneumolysin (cytotoxic-inhibits cilia function, lyses RBC, activates classical complement pathway, stimulates
cytokines…tissue damage and purulent inflammation)
Hydrogen Peroxide (tissue damage)
Surface protein adhesions (CbpA)
Neuraminidase
IgA protease
Peptidoglycan (activates alternate complement pathway, cytokine release)
Transformation (antibiotic resistane-uptake of new DNA)
Intracellular Invasion (invade cell and hide form immune system)
N) Epidemiology: colonization preceeds disease; spreads by droplets
O) Disease type & spectrum: causes ear infections in kids; cause of meningitis; common cause of community
acquired pneumonia…results in bacteremia
P) Means of detection or diagnosis: culture on sheep blood agar; catalase neg; Optochin Test ( optochin
susceptible = Ab-type compound selectively inhibits pneumococcus, but not other  hemolytic streps); bile
solubile
Q) Unique features: capnophile, 90 capsular serotypes; no Lancefield Ag; has species-specific C
polysaccharide
R) Prevention: adults: esp people over 65, those with functional asplenia and sickle cell disease (23 valent
polysaccharide vaccine); children: given during infancy and could have better memory and has reduced the
antibiotic resistant pneumococci carriage in kids (7 valent conjugate vaccine)
Viridans Streptococcus species (Alpha Hemolytic)
S) Gram stain rxn: gram +
T) Cell morphology: cocci in long or short chains
U) Natural habitat or reservoir: commensals of the mouth, upper resp. tract
V) Major virulence factors: hyaluronic acid capsule (produces hyaluronidase)
W) Epidemiology: not mentioned
X) Disease type & spectrum: cause of endocarditis (affects the heart valve); cause of dental caries (Strep
mutans-polymerizes dextran from glucose)
Y) Means of detection or diagnosis: culture on sheep blood agar, catalase neg;  hemolytic; can be
differentiated biochemically
Z) Unique features: capnophiles; no Lancefield Ag; low pathogenicity
Prevention: not mentioned
Gram positive bacilli
Corynebacterium diphtheriae
a. Gram stain reaction
Gram-positive
b. Cell morphology (rod, coccus, etc.)
Curved pleomorphic rods (“Chinese letters”)
c. Natural habitat or reservoir (humans, animals, water, soil, etc.)
Throat
d. Major virulence factors (how it causes disease)
Toxin acts locally on mucous
membranes of respiratory tract
e. Epidemiology (life cycle, how infection is contracted, transmissibility, etc.)
Spreads by droplet
• Adherent pseudomembrane of
fibrin, bacteria, epithelial and
phagocytic cells impairs breathing
• Toxin spreads hematogenously
to heart, CNS, and adrenals
• Cytotoxic effect mediated by
interfering with protein
biosynthesis by inactivating EF-2
f. Disease type & spectrum (relate virulence factors to manifestations)
Causes Diphtheria
g. Means of detection or diagnosis (culture, serology, biopsy, PCR, etc.)
– Loeffler's serum medium: C. diphtheriae
grows rapidly, faster than other upper
respiratory tract bacteria and its cellular
morphology is enhanced
– Potassium tellurite medium inhibits normal
flora, selects for corynebacteria.
– Elek plates containing antitoxin on paper
strips to confirm toxin production
– Limited to public health labs.
h. Unique features to distinguish from others (structure, biochemicals, etc.)
Curved pleomorphic rods (“Chinese letters”)
i. Prevention (public health measures, vaccines, antibiotics, etc.)
• Protect by vaccination (toxoid)
Bacillus anthracis
a. Gram stain reaction
Gram positive
b. Cell morphology (rod, coccus, etc.)
Rods..often in chains, usually with spores
c. Natural habitat or reservoir (humans, animals, water, soil, etc.)
Animal products
d. Major virulence factors (how it causes disease)
Cutaneous inoculation or respiratory
e. Epidemiology (life cycle, how infection is contracted, transmissibility, etc.)
Usually cutaneous inoculation of anthrax spores- slow healing ulcer, bacilli spread to lymphatics and
bloodstream
f. Disease type & spectrum (relate virulence factors to manifestations)
Cutaneous = 20% mortality if untreated
Respiratory anthrax usually fatal even with treatment
g. Means of detection or diagnosis (culture, serology, biopsy, PCR, etc.)
Nonmotile, May be identified with FA staining and susceptibility to specific (“gamma”) bacteriophage
h. Unique features to distinguish from others (structure, biochemicals, etc.)
Poly-D-glutamic acid (protein) capsule
Three component exotoxin
i. Prevention (public health measures, vaccines, antibiotics, etc.)
Made from avirulent, nonencapsulated B. anthracis strain
Requires series of injections and annual boosters
Used in military
Bacillus cereus
a. Gram stain reaction
Gram positive
b. Cell morphology (rod, coccus, etc.)
Rods, usually with spores
c. Natural habitat or reservoir (humans, animals, water, soil, etc.)
d. Major virulence factors (how it causes disease)
Endospores
e. Epidemiology (life cycle, how infection is contracted, transmissibility, etc.)
Contracted through endospores found in infected food (usually canned goods)
f. Disease type & spectrum (relate virulence factors to manifestations)
Food poisoning
g. Means of detection or diagnosis (culture, serology, biopsy, PCR, etc.)
β-hemalytic, catalase positive
h. Unique features to distinguish from others (structure, biochemicals, etc.)
Endospore forming
i. Prevention (public health measures, vaccines, antibiotics, etc.)
Proper packaging of food products
Listeria monocytogenes
a. Gram stain reaction
Gram positive
b. Cell morphology (rod, coccus, etc.)
Rods with straight parallel sides
c. Natural habitat or reservoir (humans, animals, water, soil, etc.)
Animal products
d. Major virulence factors (how it causes disease)
~food borne (dairy products)
~meningitis
~neonatal infections
e. Epidemiology (life cycle, how infection is contracted, transmissibility, etc.)
~food
~neonatally
f. Disease type & spectrum (relate virulence factors to manifestations)
~food borne (dairy products)
~meningitis
~neonatal infections
g. Means of detection or diagnosis (culture, serology, biopsy, PCR, etc.)
Catalase-positive
Motile (positive motility test)
Esculin positive
β hemolytic
h. Unique features to distinguish from others (structure, biochemicals, etc.)
i. Prevention (public health measures, vaccines, antibiotics, etc.)
Nocardia spp.
a. Gram stain reaction
Gram positive
b. Cell morphology (rod, coccus, etc.)
Branching rod
c. Natural habitat or reservoir (humans, animals, water, soil, etc.)
d. Major virulence factors (how it causes disease)
e. Epidemiology (life cycle, how infection is contracted, transmissibility, etc.)
f. Disease type & spectrum (relate virulence factors to manifestations)
g. Means of detection or diagnosis (culture, serology, biopsy, PCR, etc.)
~partially acid fast (so sometimes confused with the truly acidfast myco bacteria)
h. Unique features to distinguish from others (structure, biochemicals, etc.)
i. Prevention (public health measures, vaccines, antibiotics, etc.)
Gram-negative bacilli
Enterobacteriaceae
E. coli
a.) gram negative bacilli (pink to red color).
b.) Cell morphology: rod
c.) Normal flora of mammals and birds. Most common Enterobacteriaceae is in the gut.
d.) Most strains are harmless and common in humans. However all pathogenic strains have fimbriae that allow them
to bind to certain intestinal epithelial cells. They also produce toxins that cause gastrointestinal disturbances
(diarrhea). It has large virulence plasmid related to Shigella virulence plasmid.
e.) E. coli is present in fecal matter and can be transmitted through uncooked meat, contaminated water, and from
person to person if hygiene or hand washing habits are inadequate. The life cycle of E. coli in infected humans is 510 days in those with a healthy immune system.
f.) E. coli is one of the Enterobacteriaceae. It is motile with peritrichous flagella and is a facultative anaerobe. One
of the most common and easily cultured bacteria.
g.) E. coli is a facultative anaerobe that is easily cultured on an agar gel. E. coli is oxidase negative, nitrate positive,
Catalase positive, lactose positive, indole positive, and is motile.
h.) indole positive test distinguishes E. coli from other Enterobacteriaceae.
i.) Contracting E. coli is best prevented by thoroughly cooking meat, drinking treated water, washing hands, and
good personal hygiene. Antibiotics are used to treat pathogenic E. coli after symptoms appear.
Klebsiella Pneumonia
a.) Gram Stain: gram –
b.) Cell Morphology: rod
c.) Natural Habitat: humans GI tract
d.) Major Virulence Factors: Capsule, nonmotile,  lacatamase
e.) Epidemiology: Predisposing factors include: hospitalization, respirator, increased age, aspiration of oral
secretions, alcoholism, Diabetes mellitus, Chronic bronchopulmonary disease
f.) Disease Type and Spectrum: Pneumonia (non-purulent bloody sputum, necrosis/abscess) septicemia, UTI,
meningitis
g.) Means of Detection or Ds: Lactose + (MacConkey and XLD agar), resistant to ampicillin/carbenicillin, sensitive
to cephalosporins
i.) Prevention:
Proteus Mirabilis
a.) Gram Stain: gram –
b.) Cell Morphology: bacillus
c.) Natural habitat: Human intestinal tract, soil, water, plants
d.) Major Virulence Factors: tetracycline, ampicillin, & cephalosporin resistant
e.) Epidemiology: Community acquired infection
f.) Disease Type & Spectrum: UTI, wound infections, pneumonia, septicemia
g.) Means of Detection or Ds: Urease +, H2S +, Lactase –
h.) Unique Features: Highly MotileSwarm
i.) Prevention:
Salmonella Enterica
a.) Gram Stain: gram –
b.) Cell Morphology: rod
c.) Natural Habitat: humans GI tract, animals (meat & dairy), water
d.) Major Virulence Factors: adhesions, endotoxins
e.) Epidemiology: Fecal-Oral, Sanitary Food preparation
f.) Disease Type and Spectrum: Typhoid fever (enteric fever), Food poisoning (entercolitis), bacteremia
g.) Means of Detection or Ds: lactose -, most H2S +
i.) Prevention: proper food preparation
Shigella spp.
4 Named species: Shigella Dysentery, Shigella Flexneri. Shigella Boydii, Shigella Sonnei
a.) Gram Stain: gram –
b.) Cell Morphology: rod
c.) Natural Habitat: intestinal tract of humans
d.) Major Virulence Factors: endotoxins, cytotoxin, Shiga toxin
e.) Epidemiology: Fecal-Oral, Sanitary Food preparation
4 F’s of transmission: feces, fingers, food, flies, water
f.) Disease Type and Spectrum: Dysentery
h.) Unique Features: nonmotile
g.) Means of Detection or Ds: lactose-, no gas formed from glucose, H2S i.) Prevention: Sanitation, hand washing, antibiotics: ampicillin & bactrim
Yersinia Pestis
a.) Gram Stain: gram –
b.) Cell Morphology: bacillus
c.) Natural habitat: rodents
d.) Major Virulence Factors: Ca2+ dependent, endotoxin, capsule
e.) Epidemiology: Pneumonic transmission person to person, or cat to person
Bubonic transmission is from the bite of a flea
f.) Disease Type & Spectrum: Bubonic & Pneumonic Plague
g.) Means of Detection or Ds: Fluorescent Ab
i.) Prevention: rodent and flea control
Yersinia Entercolitica
a.) Gram Stain: gram –
b.) Cell Morphology: bacillus
c.) Natural habitat: soil, water, GI tract of animals
d.) Major Virulence Factors: Ca2+ dependent, endotoxin, capsule
e.) Epidemiology: transmission through food, water, and person to person contact
Fecal-oral route
f.) Disease Type & Spectrum: Enteric Infection, Mucosal ulcerations in ilium, inflamed mesenteric nodes,
abdominal pain, diarreha
g.) Means of Detection or Ds: stool sample
i.) Prevention: Sanitary Food Preparation, antibiotics: penicillin & tetracyclines
Other Gram-negative bacilli
Pseudomonas aeruginosa:
a. gram negative
b. bacillus
c. no natural habitat
d. 1. motile 2. hemolysin 3. collagenase 4. elastase 5. main virulence factor is that p.a. has endotoxin A which
blocks protein synthesis
e. Patients must be IMMUNOCOMPROMISED TO GET THIS INFECTION
f. Pneumonia: CYSTIC FIBROSIS
Sepsis: bandages on BURN PATIENTS
Endocarditis: IV DRUG USERS
Corneal infections: Contact lens wearers
g. detect with culture
h. smells like GRAPES, is beta hemolytic on Blood agar, and produces bluish-green pigmented colonies
i. broad spectrum antibiotics are used to treat
Helicobacter pylori
a.
b.
c.
d.
e.
f.
g.
h.
i.
gram negative
curved bacillus
no natural habitat
no virulence factors or toxins
people whom regularly insult there stomach with EtOH, and smoke are prone to infections
causes PEPTIC ULCERS, chronic gastritis, and PROSTATE CANCERS
detect with dark field microscopy or a breath test which functions similar to a PCR
know PEPTIC ULCER
treat with antibiotics
Vibrio cholera
a. gram negative
b. short COMMA looking bacillus
c. no natural habitat
d. fecal-oral transmission, drinking water with sewage in it
e. 1. motile 2. mucinase 3. fimbriae 4. NONINVASE
f. CHOLERAGEN: an enterotoxin increases levels of cAMP which leads to electrolyte secretion into the GI
which causes water to follow
g. Grows as flat, YELLOW COLONIES, on TCBS agar
h. Causes RICE WATER STOOLS (aka severe diarrhea)
i. Treat with doxy. and quinolones. Proper sewage treatment prevents outbreaks.
Haemophilus influenza
a. gram negative
b. bacillus (coccobacillus, almost looks like diplococci)
c. found in HUMANS only
d. has 1 of 6 capsules, b capsule is the most virulent; pilli for attachment; and IgA protease
e. transmitted thru respiratory drops
f. major cause of MENINGITIS in young kids between 6 mo and 3 y/o, also causes Acute EPIGLOTTITIS in
infant, and sepsis which can lead to SEPTIC ARTHRITIS in infants
g. detected with cultures on CHOCOLATE AGARS, only grows in the presence of NAD+ and HEMIN (X and
V), does not grow on Blood Agar
h. the capsule b (b=bad) is tough for infants to make antibodies against thus as people get older and their
immune system matures Haemophilus inf. Becomes less of a problem
i. in 1991 they came out with a vaccine against the capsule b strain and thus have dramatically reduced the
number of infections
Campylobacter spp.
a. gram negative
b. bacillus
c. habitat is wild and domestic animals, poultry
d. 1. motile 2.invasive 3. main virulence factor is an enterotoxin which acts the same way as choleragen 4.
cytotoxin which destroys cells
e. 1. uncooked meat 2. unpasteurized milk 3. fecal oral
f. Causes bloody or secretory diarrhea
g. KEY way differentiate from salmonella typhi is that campy grows at 42 degrees Celsius, and salmonella does
not
h. Culture with a CAMPY plate
i. Proper food preparation and sewage treatment are preventative, treat with erythromycin
Bordetella pertussis
a. gram negative
b. bacillus, spore forming
c. reservoir is man
d. 1. capsule 2. beta-lactamase 3. FHA: a pilli that allows the org. to attach to ciliated epithelial in bronchi
e. Pertussis exotoxin cause increased G proteins which leads to increased cAMP resulting in increased
sensitivity to histamine, release of insulin, and number of wbcs; the org also has cytotoxic effect on epis and
weakens wbcs
f. Culture on BORDET-GENGOU media (has penicillin in the agar), also can detect with an ELISA assay, or
direct microscopy
g. WHOOPING COUGH is the main manifestation, we get a PPT vaccine which has made bp a mute point.
Infants before the age of one can’t get the vaccine so they are at risk. Adults can get whooping cough from
the vaccine
Legionella pneumophilia
a. gram negative
b. bacillus
c. reservoirs are man, cooling systems, hot tubs, and water fountains
d. 1. capsule 2. motile 3. hemolysin
e. Immunocompromised people are at risk, likes to invade alveolar cells
f. Causes PONTIAC FEVER which is acute and has flu like symptoms, and a chronic condition called
LEGIONAIRES’ disease which manifests as pneumonia
g. Detect with CHARCOAL YEAST EXTRACT AGAR
h. L-cysteine is critical for growth
i. Regular chlorination of pools, and testing of cooling systems in hospitals is preventative of an outbreak
Acinetobacter
Not a lot on this organism, causes some lung infections if I remember right. Rods appear to branch on gram stain.
Gram-negative cocci
Chlamydiae
Mycoplasmas
Spirochetes
Rickettsiae
Mycobacteria
(I’m not sure which organism this if for??)
a. gram stain- not the best, identified by acid fast staining
b. cell morphology-red rods with acid fast stain
M. Tuberculosis flaky white dry colonies
M. avium non-pig colonies
c. natural habitat-obligate aerobes
M. tuberculosis humans are the only natural host
M. avium soil and water
M. leprae- humans and armadillos
d. virulence factor- mycolic acids
e. epidemiology- slow growing
M. tuberculosis- 90% of infected persons will not develop TB, survives drying, susceptible to UV radiation,
resistant to many disinfectants need to use chlorine to kill, killed by pasteurization, transmitted by infectious
droplets 5-200.
M. avium transmission respiratory or GI tract
M. leprae transmitted by inhalation or skin contact with contaminated respiratory secretions
f. disease type and spectrum
M. leprae- leprosy or Hansen’s disease, organism lives in nerves in cooler parts of the body
g. means of detection and diagnosis
M. tuberculosis diagnosed with AFB smear, TB skin test, Cehst radiograph, AFB culture
M. leprae detection AFB stain of nasal secretions, lepromin skin test, prevention vaccine under development
h. unique features mycobacteria are acid fast stained!
i. prevention-M. tuberculosis decrease exposure, chemoprophylaxis, BCG vaccine
Anaerobic Bacteria
In general (the same for all anaerobes):
Diagnosis:
Clinical signs
1. foul smelling discharge (because of short chain fatty acids as an end product)
2. proximity to a mucosal surface
3. gas in tissue
4. abscess formation
Gram stain (may be helpful in the establishment of a mixed infection or the presence of closdtricia in wounds
Culture – sample collections and transport are critical; require complex medium supplemented with hemin, Vit.
K, and/or blood; media should include antibiotics(aminoglycoside) to suppress facultative anaerobes; incubation
and work up performed in CO2 in nitrogen/hydrogen mix
Treatment:
Surgical drainage of closed abscess
Mixed infections – cover for both aerobic and anaerobic component
Metronidazole, penicillin G, Clindamycin (B. fragilis and P. melaninogenica are penicillin resistant);
Aminoglycoside are not effective
Toxin mediated diseases – antitoxin and antibiotics if active infection vs. intoxification
Pathogenesis:
Synergy with facultative organisms
Facultative bacteria such as Enterobacteriaceae function to reduce the oxygen content in the tissue
Beta-lactamase production
Diseases:
Vincent Agina – synergistic infection of Fusobacterium, Bacteroides, and Treponeme (gums receding away
from teeth)
Necrotizing Fasciitis – usually associated with Streptococcal dz, but sometimes caused by anaerobes
Bacteroides fragilis
a.) Gram-negative bacilli (rods), non-sporeforming pleomorphic rods (take on different appearances)
c.) Natural habitat is normal human floral, mainly associated with the colon
d.) Causes disease by f.) Diseases – Necrotizing Fasciitis:
g.) Means of Detection or Diagnoses – bile(+) and esculin (+)
h.) Unique features – capsule is antiphagocytic, resistant to penicillin, pleomorphic
i.) Prevention – antibiotic therapy with surgical inervention
Clostridium tetani
a.) Gram-positive bacilli, sporeforming; looks like lollipops because of the terminal spores larger than vegetative cell
c.) Natural habitat – ubiquoitus, found in soil
d.) Causes disease by – toxin production
f.) Diseases – Tetanospasmin (causes continuous firing of motor neurons); “Lock Jaw” (masseter muscles are
clenched)
g.) Means of Detection or Diagnoses – presence of toxin
h.) Unique features – “lollipop” appearance
i.) Prevention – vaccine (DPT-given when young)
Clostridium perfringes
a.) Gram- positive bacilli, sporeforming; “boxcar” like
c.) Natural habitat – in humans intestinal tract and in the soil
d.) Causes disease by – Phospholipace C (alpha-toxin):gas gangrene, destrpys WBCs, platelets, and a bunch of other
host cells; enterotoxin:Food poisoning
f.) Diseases – Gas gangrene, food poisoning
g.) Means of Detection or Diagnoses – Etest Susceptibility Testing, on agar the colony will be surrounded by a hazy
circle – double zone of hemolysis (due to all the toxins made); Nagler Test (has antitoxin only on one side of the agar
plate)
h.) Unique features – “boxcar”
i.) Prevention –
Clostridium botulinum
a.) Gram- positive bacilli, sporeforming
c.) Natural habitat – ubiquoitus, found in soil worldwide
d.) Causes disease by – ingestion of preformed neurotoxin except for wound and infant botulism; mostly food-borne
dz
f.) Diseases – neurotoxin blocks muscle contraction, infant botulism (most common form of botulism in US, form
ingestion of environmental material i.e. honey)
g.) Means of Detection or Diagnoses – find toxins in food or patient feces or serum
h.) Unique features –
i.) Prevention – no vaccine, metrodiazole, antitoxin
Clostridium difficile
a.) Gram- positive bacilli, sporeforming
c.) Natural habitat – human intestinal tract
d.) Causes disease by – antibiotics for another disease wipes out all of the normal flora, except C. difficile (which is
typically resistant); toxin
f.) Diseases – Pseudomombranous colitis/antibiotic related diahrea
g.) Means of Detection or Diagnoses –
h.) Unique features –
i.) Prevention –
Actinomyces spp.
a.) Gram-positive bacilli
c.) Natural habitat – in humans associated with oral, respiratory and female genital tract infections (IUD); common
in soil as well
d.) Causes disease by –
f.) Diseases – causes extensive soft tissue involvement, crosses tissue plane and involving multiple organ systems;
form sulfur granules (clumps of organisms that have host material deposited on them, 1-2 mm, if crushed and gram
stain (-) filamentous branching structures)
Dacrocryocystitis – blockage of tear ducts, must remove sulfur granules
Actinomycosis (“Lumpy Jaw”) – poor hygiene, allowing abscess formation on the roots of teeth,
destruction of root and bone, if left alone will it will make its way exterior
g.) Means of Detection or Diagnoses – slow growing and are difficult to isolate; presence of sulfur granules
h.) Unique features – sulfur granules, not aerotolerant
i.) Prevention –
Fusobacterium spp.
a.) Gram-negative bacilli, pleomorphic nonspore forming rods; long, thin, some with pointed ends
c.) Natural habitat – present in the human upper respiratory tract and intestinal tract (mouth, colon)
d.) Causes disease by – usually present in mixed infections but may be the sole agent
f.) Diseases – pulmonary abscesses
g.) Means of Detection or Diagnoses –
h.) Unique features – rods with pointed ends
i.) Prevention –
Diagnostic Tests
Gram stain: allows colorless bacteria to be visualized in clinical specimens and in culture media; provides a basis
for bacterial classification; allows bacterial cell morphology to be viewed. Gram positive: stain purple. Gram
negative: stain pink.
After preparation of Smear,
1. Flood slide with crystal violet solution (adheres to the peptidoglycan of the cell wall) and allow it to remain on the
slide for 60 seconds. Rinse slide with water.
2. Flood slide with iodine solution (serves as a mordant by increasing the affinity of the dye for the bacterial cell) and
allow to remain for one minute. Rinse slide with water.
3. Rinse slide with 95% alcohol (washes away the non-adherent to cell wall CV solution) until the solvent flows
colorlessly from the slide (about 2 seconds).
4. Counterstain the bacterial smear on the slide with safranin (stains the gram negative bacteria which have thin
peptidoglycan walls that have been rinsed clean of the crystal violet) for 30 seconds, then wash slide with water.
Kinyoun acidfast stain
Auramine-Rhodamine Stain
Alpha, Beta, Gamma Hemolysis
Pattern of hemolysis is used to classify the group Strep. Bacteria can hemolyze red blood cells by the production of a
variety of different toxins called streptolysins.
You can classify hemolysis into 3 different types:
1. alpha: means partial hemolysis; you see a green discoloration of the blood agar
2. beta: means complete hemolysis; completely lyse the RBCs and can see through the agar plate so it will look clear
3. gamma: means non-hemolytic
Catalase Test: The catalase test involves adding hydrogen peroxide to a culture sample or agar slant. If the bacteria
in question produce catalase, they will convert the hydrogen peroxide and oxygen gas will be evolved. The evolution
of gas causes bubbles to form and is indicative of a positive test.
Coagulase Test: The coagulase test is another method for differienting between pathogenic and non-pathogenic
strains of Staphylococcus. Bacteria that produce coagulase use it as a defense mechanism by clotting the areas of
plasma around them, thereby enabling themselves to resist phagocytosis by the host's immune system. The sample in
question is usually inoculated onto 0.5 ml of rabbit plasma and incubated at 37 degrees celsius for one to four hours.
A positive test is denoted by a clot formation in the test tube after the allotted time.
Optochin test
Bacitracin Test: The bacitractin sensitivity test identifies whether an organism is susceptible or resistant to the
antimicrobial bacitracin. A blood agar plate is streaked for confluent growth with the organism to be tested, and a
paper disk that has been infused with bacitracin is applied to the surface of the agar. Bacitracin acts by interrupting
the formation of the cell wall. The bacitracin test is useful for differentiating β-hemolytic Group A streptococci
(susceptible) from β-hemolytic non-Group A streptococci (resistant). This is important because most streptococcal
diseases are caused by Group A streptococci.
Novobiocin test: used to distinguish Staphylococcus saprophyticus (which is novobiocin resistant) from other
coagulase negative staphylococci found in humans. Used for urinary tract isolates.
Oxidase test- Used as an aid in recognition of Neisseria coloniesin cultures for gonococci and in the identification of
certain non-fermentative gram negative rods. Aeromonas, Campylobacter, Haemophilus, Moraxella, Neisseria,
Pseudomonas, and Vibrio are oxidase positive. Enterobacteriaceae are oxidase-negative.
PYR test
Indole test
Bile esculin test
Application: Used in conjunction with 6.5% sodium chloride broth to distinguish enterococci from streptococci
Medium: A nutriest agar containing 40% bile, which is inhibitory to most bacteria, and 0.1% esculin
Procedure: Inoculate the surface of the slant heavily with the organism to be tested; with the same inoculum, seed a
tube of sodium chloride (6.5%) broth. Examine after overnight incubation.
Results: Black means positive (Enterococcus is an example of a positive result)
6.5% NaCl test: used in conjunction with the Bile-esculin test usually-which distinguishes Enterococcus from
Streptococcus (except Group D strep). Growth in bile and hydrolysis of esculin to produce black in nutrient agar is a
positive test for either Enterococcus or Group D streptococcus. NaCl test is used to distinguish the Enterococcus
from the Group D streptococcus. A positive test for Enterococcus is growth on the 6.5% NaCl
CAMP test- (arrow head test) Best for distinguishing group a streptococci from group B streptococci. Group B strep
secrete a protein (CAMP factor) that enhances the hemolytic action of staphylococci beta lysine on blood agar. To
perform, S. aureus is streaked on a blood agar plate. The unknown organism is streaked perpendicular to but not
touching the staphylococcus and the plate is incubated overnight. An arrow head of clear hemolysis indicates a group
B streptococcus.
Motility Test- pierce semi-solid agar and look to see if organism spreads away from intial inoculation. Ex. Listeria
monocytogenes is positive for motility
Sheep Blood Agar - Purpose Blood agar is used both as an enriched medium for growing fastidious bacteria and as a
differential medium. Exotoxins called hemolysins cause lysis of the red blood cells. The degree of the hemolysis is
an especially useful tool for identification of many of the Gram positive cocci.
MacConkey agar
XLD agar- Xylose lysine desoxycholate agar is a selective, differential medium containing sucrose, lactose, and
xylose as carbohydrate sources. Lysine is added to detect the lysine decarboxylase reaction. Bile salts in low
concentration selectively inhibit gram-positive organisms but not coliforms. Sodium thiosulfate and ferric
ammonium citrate are added to permit detection of hydrogen sulfide gas production. Phenol red is included as a pH
indicator. Salmonella colonies will appear red due to hydrogen sulfide following decarboxylation of lysine and
contain black centers due to hydrogen sulfide production in the presence of ferric ammonium citrate. Shigella
colonies will appear colorless or red. E. coli and Enterobacter colonies will appear yellow due to carbohydrate
utilization.
Chocolate agar- This medium is prepared in the same mannor as blood agar except that the medium is heated to 80
degrees C for 15 minutes or until the color is chocolate brown. This process lyses the red blood cells, thus releasing
hemoglobin and other growth factors needed by several fastidious bacteria. This heating process also coagulates
proteins in blood, helping absorb fatty acids and amino acids that are inhibitory to some bacteria such as Neisseria
gonorrhoeae. Like N. gonorrhoeae, Haemophilus influenzae is cultivatable on chocolate agar but not on ordinary
blood agar.
Mannitol salt agar (MSA): Purpose- Mannitol salt agar is both a selective and differential growth medium. It is
used to differentiate pathogenic Staphylococcus species from nonpathogenic members ofthe genus Micrococcus. an
organism can be salt tolerant or use mannitol or both. Micrococcus is salt intolerant and Staph is salt tolerant.
Muellar Hinton agar:
Has starch and casamino acids, NISSERIA grows well on this agar
Thayer Martin agar:
Selects for Nisseria Gonorrhea and Meningococci species, growth = n.g. or n.m.
Sabouraud Dextrose Agar:
Contains dextrose, nitrogen, and acid pH which conducive to Fungi, selects for fungi
Lowenstein Jensen agar:
Selects for Mycobacteria, very nutrient rich
Selective media:
Media such as chocolate agar and MacConkey agar which contain ingredients which cause certain bacteria to be
inhibited while other grow
Differential media:
Media that has certain indicators, such as dyes, which stain organisms the utilize one ingredient a certain color,
where as other organism would stain a different color. API test strips are differential medias.
Enriched media:
Media in which nutrients have been added. Brain Heart Infusion agar is an example. Grows all sorts of organisms.
Nutritive media:
About the same as enriched.
Oxidation reaction:
Reaction that takes place under normal atmospheric oxygen levels. Oxygen is required by the bacteria for
metabolism.
Fermentation reaction:
Reaction takes place in the absence of oxygen, or decreased amounts. Seen in tubes where oil placed on top of
media.
API Biochemical Strip:
Set of medias which are inoculated and incubated to identify gram negative bacteria. Read the reactions which
correspond to a nine digit number which is used to look up the organism.
Agar disk diffusion test:
Used to check for gross antibiotic suseptability. Streak the plate with one organism and drop antibiotic disks to
check for inhibition of growth.
Agar gradient diffusion test:
The E test where you streak a plate with organism and drop a gradient antibiotic strip that quantitatively gives you
the MIC.
Agar dilution test:
Looking for what dilution of antibiotic inhibits the growth of the organism. Do a serial dilution to find out the least
amount antibiotic needed to inhibit growth.
Microbroth Dilution Test/Minimum Inhibitory Concentration (MIC)
Dilution methods are used to determine MICs of antimicrobial agents, and they are reference methods for
determining the susceptibility of organisms to certain antibiotics.
To find the minimum inhibitory concentration for a microbroth dilution test, defined amounts of bacteria are
inoculated into a nutrient broth containing a dilution series of antibiotic concentrations
In this case, Staph aureus and tetracycline were used.
This microbroth dilution test shows that, bacterial growth occurred at a Tetracycline concentration of 0.8 mcg/ml but
not at 1.6 mcg/ml. Thus, the minimum inhibitory concentration is 1.6 mcg/ml.
The E- test, where a commercially-prepared strip creates a gradient of antibiotic concentration when placed on an
agar plate, can also be used to determine (MIC) of 4.
Satellitism – this test is used to differientiate Haemophilus species on the basis of their respective requirements for X
(heme) and V (NAD) factors. The medium used is Trypticase Soy Agar (TSA) plate. The procedure is to streak a
complete agar plate with the organism in question and place paper strips containing X, V and XV factors on the plate
using flamed forceps. The plate in incubated for 24-48 hours in a CO2 jar and the pattern of growth of the unknown
organism in relation to the factor-containing strips of paper is observed. Growth only around X strip indicates a
requirement for heme. Growth only around the V strip indicates a requirement for NAD. Growth only around XV
strip indicates a need for both.
X and V Factor Test
You can use this test to characterize different types of Haemophilus, because it requires exogenous hemin (X factor)
and/or NAD (V factor). X and V factor are found in chocolate agar.
Sterility test with biological indicator – Autoclaving is the method of choice for sterilization of instruments that are
not damaged by heat or moisture. One of the most common testing procedures is the use of an ampule containing
spore suspensions of Bacillus stearothermophilus in culture medium as a biological indicator of the effectiveness of
the sterilization process. The ampules have been standardized so that exposure for at least 15 minutes at in a proper
autoclave environment results in sterilization. If sterilization is incomplete the purple medium turns yellow upon
incubation due to germination of spores and growth of the organism.
Enzyme immunoassay
Complement fixation
Latex Agglutination-is a rapid assay for detecting the antibody or soluble antigen. Virus specific Ab cause latex
particles with viral antigens to clump. Also can have latex coated with antibodies to detect specific antigens
Dark Field Microscopy-Darkfield microscopy is a specialized illumination technique that capitalizes on oblique
illumination to enhance contrast in specimens that are not imaged well under normal brightfield illumination
conditions.
Terminology
Abscess
Active immunization
Adaptive immunity
Antibiotic: an agent that can be naturally occurring, partially or completely synthetic that selectively inhibits the
growth of microorganisms at low concentrations (ex. Penicillin)
Antibiotic Synergy
Antibiotic synergy is the combination of two or more antibiotics so that their combined effect is greater than the sum
of their individual effects.
Antibiotic antagonism
Antiseptic: an agent used to inhibit or eliminate microbes on skin or other living tissue (ex. alcohol, iodine,
chlorhexidine)
Autoclave - a pressurized device that enables the heating of aqueous solutions up to temperatures above the boiling
point of water. This means that bacteria, viruses, fungi, and spores are inactivated.
Bactericidal-preventing infection by inhibiting the growth or action of microorganisms; destroying bacteria
Bacteriostatic- Inhibiting or retarding the growth of bacteria
Bacteriophage- A virus that infects and lyses certain bacteria
Beta lactamase
Beta lactamase inhibitor- Inactivate Beta-lactam antibiotics. Beta-Lactam Ab attack bacterial cell wells by
inhibiting formation of cross-links by petidoglycans. Beta lactamase inhibitor is the bacteria’s defense mechanism
against Beta-lactam Ab.
Biotype are organisms sharing a specified genotype or the genotype (or peculiarities) so shared
Capsule- Some species of bacteria feature a protective covering, the capsule, which is composed either of proteins,
or more commonly of polysaccharides, a class of complex carbohydrates. The capsule prevents desiccation of the
bacteria and helps prevent phagocytosis by larger microorganisms and the white blood cells of invaded host
organisms. Additionally, capsules help the bacteria adhere to the host substrate and assist in warding off attacks by
viruses (bacteriophages). There are some indications that the capsule and its associated biofilms are toxic or
chemically inhibitory to hosts' defenses, thus aiding the disease mechanism. For some pathogenic bacteria, such as
the common Escherichia coli and Streptococcus pneumoniae, the virulence of a strain is dependent upon the
function of the capsule. In contrast, non-encapsulated mutants of these prokaryotes are avirulent.
A carbuncle is an abscess larger than a boil, usually with one or more openings draining pus onto the skin. It is
usually caused by bacterial infection
Cell Wall: The bacterial cell wall is a unique structure which surrounds the cell membrane. Although not present in
every bacterial species, the cell wall is very important as a cellular component. It provides shape, Barrier (osmotic
resistance), comprised of highly cross linked peptidoglycan, primary target of many antimicrobials, and basis for
gram stain.
Cell Membrane: I would hope everyone knows what this is.
Cellular immunity
Cellulitis- The word cellulitis literally means inflammation of the cells. It generally indicates an acute spreading
infection of the dermis and subcutaneous tissues resulting in pain, erythema, edema, and warmth. In individuals with
normal host defenses, the most common causative organisms are group A streptococci and Staphylococcus aureus.
Cellulitis in infants may present as sepsis, most commonly caused by group B streptococci. In immunocompromised
hosts, gram-negative rods or fungi may cause cellulitis, though fungal cellulitis is rare. Wounds occurring after
exposure to fresh water may be caused by Aeromonas hydrophila, a gram-negative rod. Pneumococcus may cause a
particularly malignant form of cellulitis, typically in an immunocompromised host, and frequently is associated with
tissue necrosis, suppuration, and blood stream invasion.
Cistron
Codon: a sequence of 3 bases in messenger RNA that encodes a specific amino acid
Conjugation- is the transfer of genetic material between bacteria through cell-to-cell contact. (as opposed to
transformation or transfection). Bacterial conjugation is often regarded as the bacterial equivalent of sexual
reproduction or mating. However, it is not actually sexual, it is merely the transfer of genetic information from a
donor cell to a recipient.
Colonization- Organisms that colonize humans (whether for a short period such as hours or days [transient] or
permanently) do not interfere with normal body functions.
Commensal
Complement: A complex of proteins in the blood serum that interacts sequentially with specific antigen antibody
complexes.
Cytokine
Disinfectant: an agent used to destroy microbes on inanimate objects (ex. phenols, formaldehyde, chlorine bleach)
DNA hybridization
Empyema
Endotoxin: Part of the outer portion of the cell wall (lipid A) of most gram-negative. Endotoxin is released on
destruction of the cell wall.
Enterobactin – an iron chelating siderophore released by enteric bacteria (like E. coli and Salmonella) to bind iron
and deliver it to the pathogen; if a bacterium cannot make enterobactin, then it cannot grow in the gut
Enterotoxin: An exotoxin that causes gastroenteritis, such as those produced by Staphylococcus, Vibrio, and
Escherichia.
Exotoxin
Extended Spectrum Beta Lactamase
Frameshift mutation
Furuncle
Gene
Generalized transduction
Genetic code
Flagella
H antigen
Herd immunity: the resistance to invasion and spread of an infectious agent in a group or community, based on the
resistance to infection of a high proportion of individual members of the group.
Homolgous recombination – the exchange of corresponding stretches of DNA between two sister chromosomes.
Humoral Immunity – B cell immunity; antibody immunity; refers to antibody production and all of the processes
that go with it, including cytokine production, memory cell production, isotype switching, etc.
Innate immunity—immunity to disease that occurs as part of an individual's natural biologic makeup; mediated by
cells capable of immune activity without being stimulated by immunization and without antigen specificity. It also
includes local barriers to infection such s skin, stomach acid, mucous, enzymes in tears and saliva, and skin oils.
Interferon- Any of a group of glycoproteins that are produced by different cell types in response to various stimuli,
such as exposure to a virus, bacterium, parasite, or other antigen, and that prevent viral replication in newly infected
cells and, in some cases, modulate specific cellular functions
Interleukin-any of various compounds of low molecular weight that are produced by lymphocytes, macrophages,
and monocytes and that function especially in regulation of the immune system and especially cell-mediated
immunity
K antigen-found in E. coli, renders them resistant to phagocytosis
Lipid A: contains toxic properties of LPS
Lipopolysaccharide (LPS): an endotoxin and its toxic properties include toxic shock, fever, leukopenia,
hypotension, acidosis, DIC, and death
LPS makes up the outer leaflet of the Outer Membrane (OM) in gram negative bacteria
LPS Structure:
Outer Mem brane—Lipid A—Core Polysaccharide—O Ag (Ab to LPS and directed against O Ag)
Lysogeny: when you have a toxin that is encoded on a bacterial virus that infects the strain of bacteria which allows
it to encode, transcribe, and translate the toxin there. (seen with scarlet fever in group A strep)
Acquired drug resistance—changes in bacterial DNA through either mutations or gene transfer that render
antibiotics less effective
Lysozyme: an enzyme present in tears, saliva, and sweat that kills bacteria. It is the body's main defense against eye
infections.
Macrophage: a phagocytic leukocyte in the tissue. An APC.
Minimal inhibitory concentration (MIC): Refers to an antibiotic concentration that is just enough to stop the
growth of an organism but not kill it. Clinically more important than MCC, as long as host as a effective immune
system.
Minimal cidal concentration (MCC): Refers to antibiotic concentration that is strong enough to kill 99.9% of the
organism.
Missense mutation: A substitution of one base in a DNA for another that results in a different amino acid in a
polypeptide. A missense mutation is readable, a NONSENSE mutation is the same as a missense, but it can not be
read and so polypeptide formation is stopped.
Nonhomologous recombination: Chromosome deletions, inversions, and translocations are often considered the
result of nonhomologous recombination. (Read through bacterial pathogenesis transcripts for more details.
Null mutation: A change in a gene that leads to it not being transcribed into RNA or translated into DNA.
Normal flora: Bacteria that is normally found on or in your body. Is not normally a deficit to your overall health,
and in some instances is very beneficial (ex. Vitamin K is made by bacteria in your gut.)
Nosocomial infection: Infection acquired as result of being in a hospital.
O antigen: Polysaccharide component of LPS, is the binding site for bacterial viruses; also has endotoxic activity so
it’s involved in toxic shock.
operon: A genetic functional unit that controls production of a messenger RNA; it consists of an
operator gene and two or more structural genes located in sequence in the cis position on one chromosome.
opportunistic pathogen: an organism that is capable of causing disease only when the host's resistance is lowered,
e.g., by other diseases or drugs.
Opsonization—process whereby opsonins (antibodies or products of complement activation in blood serum) make
an invading microorganisms and other cells more susceptible to phagocytosis
Osmotic Diarrhea
This type of diarrhea is due to the presence of an unabsorbable or poorly absorbable luminal osmols that exert
an osmotic pressure effect across the intestinal mucosa, resulting in excessive water output. Because the diarrhea is
caused by the solute, it tends to stop during fasting.
*Increased intra-luminal osmotic pressure
*Decreased absorption of fluid and solutes
Causes of osmotic diarrhea include disaccharidase deficiency (lactose intolerance). Giardia lamblia.
About 80% of the world population has primary lactase deficiency. Black Americans and Jews have the highest
incidence. The condition begins in childhood and cannot be outgrown.
Passive immunization: It is sometimes the case that an infectious organism or a poisonous substance can have such
a rapid deleterious effect that the victim does not have time to develop an immune response spontaneously. At such
times passive immunization with preformed antibodies can provide life-saving assistance in combating the pathogen
or poison. There is naturally aquired passive immunity, which involves the transfer of antibodies from a mother to
her infant, and then there is artificially acquired passive immunity, which involves the introduction of antibodies
(rather than antigens, which is artificially acquired active immunity) into the body. These antibodies come from an
animal or person who is already immune to the disease.
Peptidoglycan
• Backbone of N-acetyl glucosamine and Nacetyl muramic acid (transglycosylation)
• Cross-linked by peptide bridges at MurNAc (transpeptidation)
• β-lactam antibiotics prevent crosslinking
• Lysozyme (hydrolase) - cleaves backbone (M-G)
Phagocytosis: engulfment of bacteria via invagination of membrane of pathogen to make phagosome which later
fuses with lysosome to make phagolysosome. With this fusion, the pathogen is ingested via heterophagy or
autophagy.
Plasmids - Replicate in cytoplasm, independent of chromosome.
– Usually circular (borrelia = linear)
– Few to several hundred kb.
– Conjugative (F, R), antibiotic resistance, metabolic, virulence
Pili - protein. Shorter, narrower than flagella.
– Common - peritrichous; attachment
– F (sex) - single; gene transfer (conjugation; gram -)
Primary drug resistance: Primary drug resistance is a term used to identify resistance observed in HIV-infected
individuals who have never before received treatment and so presumably have been infected with a drug-resistant
strain of HIV.
Promotor
Restriction Enzymes
Specific strains of microorganisms can be distinguished by their DNA fragments produced when the DNA is cleaved
by specific restriction enzymes. Restriction enzymes recognize specific DNA sequences that have a palindromic
structure.
Secretory Diarrhea - In this type of diarrhea there is abnormal ion transport across the intestinal epithelial cells,
which results in increased secretion, decreased absorption, or both. There is no osmotic gap, and as the diarrhea is not
related to intestinal contents, it typically does not stop with fasting. A classic example of acute secretory diarrhea is
cholera, and is also seen in infections like enterotoxigenic E coli.
Serotype
Shiga Toxin- are a family of related toxins with two major groups, Stx1 and Stx2, whose genes are considered to be
part of the genome of lambdoid prophages. Shiga toxins act to inhibit protein synthesis within target cells. Common
in Shigella and E. Coli.
A Siderophore (greek for iron carrier) is an iron chelating compound secreted by microorganisms. Iron Fe3+ ions
have a very low solubility at neutral pH and therefore cannot be utilized by organisms. Siderophores dissolve these
ions as soluble Fe3+ complexes that can be taken up by active transport mechanisms. Many siderophores are
nonribosomal peptides.
Other strategies to enhance iron solubility and uptake are the acidification of the surrounding (e.g. used by plant
roots) or the extracellular reduction of Fe3+ into the more soluble Fe2+ ions.
Examples of siderophores produced by various bacteria and fungi are ferrichrome (Ustilago sphaerogena),
enterobactin (Escherichia coli), enterobactin and bacillibactin (Bacillus subtilis), ferrioxamine B (Streptomyces
pilosus), fusarinine C (Fusarium roseum), yersiniabactin (Yersinia pestis), vibriobactin ( Vibrio cholerae),
azotobactin (Azotobacter vinelandii), or pseudobactin (Pseudomonas B 10).
Silent mutation: the form of a genetic trait distinguishable at the genotypic level but not at the level of arbitrary
phenotype (i.e., the genotype changes but nothing noticeable happens.)
Specialized Transduction
SuperAntigen- class of microbial proteins that bind to Class II MHC molecules outside antigen-binding cleft and
stimulate CD4 T cells by interacting with all TCRs that display a specific variable region on the  chain
-typified by staphylococcal exotoxins
Superoxide dismutase- The enzyme superoxide dismutase, or SOD, catalyzes the dismutation of superoxide into
oxygen and hydrogen peroxide. E. coli contain a form of this enzyme. SOD is also an effective defence weapon and
Mycobacteria and Nocadia have SOD which enables them to resist the injection of superoxide by phagocytes. When
these organisms cause serious disease, it takes the body a very long time to win, and depending on the strength of the
patient the bacteria may win. Although the enzyme isn't especially fast relative to the spontaneous dismutation of
superoxide, the ability of the enzyme to provide some protection to organisms is shown by the existence of a motor
neuron disease in individuals who have point mutations in SOD and by the finding that the absence of SOD may lead
to a form of anemia. As an enzyme, SOD has particular value as an antioxidant that can help to protect against cell
destruction. It has the distinct ability to neutralize superoxide, one of the most damaging free radical substances in
nature. ALS or Lou Gehrig’s disease is related to a mutation in the SOD1 gene.
Teichoic acid: The cell walls of gram-positive bacteria contain teichoic acids, which consist primarily of an alcohol
(such as glycerol or ribitol) and phosphate. There are two classes of teichoic acids: lipoteichoic acid, which spans the
peptidoglycan layer and is linked to the plasma membrane, and wall teichoic acid, which is linked to the
peptidoglycan layer.
Because of their negative charge, (from the phosphate groups), teichoic acids may bind and regulate the movement of
cations (positive ions) into and out of the cell. They may also assume a role in cell growth, preventing extensive wall
breakdown and possible cell lysis.
Finally, teichoic acids provide much of the wall’s antigenic specificity and thus make it possible to identify bacteria
by certain laboratory tests.
Transcription
Transformation— transfer of genetic information between bacteria by means of “naked” intracellular DNA
fragments (plasmids) derived from bacterial donor cells and recombination in the recipient chromosome
Translation
Transition mutation
Transposition
Transposon
Transversion mutation
Vaccine
Virulence Factor-properties that allow pathogens to survive, spread, and cause disease
SPECIES
H. influenzae
H. parainfluenzae
GROWTH REQUIREMENTS
X (Hemin)
+
-
V (NAD)
+
+
H. hemolyticus
H. ducreyi
.
Terminology
***************
+
+
+
-